Download as docx, pdf, or txt
Download as docx, pdf, or txt
You are on page 1of 59

NATRES CASE DIGEST

SUNBEAM CONVENIENCE FOOD VS. CA The registration was opposed by Bayaua,


(181 SCRA 443) Reyes, and the Philippine Cacao and Farm
FACTS: Products.
Sunbeam Convenience Foods, Inc. is
the recipient of a Sales Patent issued by the The case went on until on 1981, 20 years after,
Bureau of Lands over two parcels of land in the Heirs of Sandoval, Heirs of Bayaua, and
Bataan. An OCT was thereby issued. The the Bureau of Lands and Bureau of Forest
Solicitor-General filed an action for reversion Development entered a compromise
on the ground that the lots were forest lands agreement, which effectively distributed parts
and therefore inalienable. of lot 7454 among the aforesaid parties and the
counsel of the Heirs of Sandoval as attorney's
CA ruled, upholding the Solicitor- fees. The compromise agreement was
General's contention. approved by the court and confirmed the title
and ownership of the parties in accordance
ISSUE: Whether or not land is alienable and with its terms.
disposable land of the State.
Having knowledge of the incident, the Solicitor
HELD: General filed a complaint before the court to
No. annul the decision rendered by the court a quo
for being void and made in excess of
Our adherence to the Regalian Doctrine jurisdiction or with grave abuse of discretion.
subjects all agricultural, timber, and mineral
lands to the dominion of the State. Thus, The Solicitor General contended that the the
before any land may be declassified from the Heirs of Sandoval et. al. did not present any
forest group and converted into alienable or evidence to support their claims of ownership
disposable land for agricultural purposes, there or registration, nor did the government
must be a positive act from the Government. agencies involve have a y authority to enter
Even rules on the confirmation of imperfect into the compromise agreement, and finally,
titles do not apply unless and until the land that he was not notified of the proceedings and
classified as forest land is released in an so had not opportunity to take part therein.
official proclamation to that effect so that it may
form part of the disposable agricultural lands of As for the Heirs of Sandoval et.al.'s contention,
the public domain. they asseverate that the land is not a public
land as the possessory information title in their
The mere fact that a title was issued by name and of their predecessors-in-interest, the
the Director of Lands does not confer any pre-war certification appearing in the Bureau of
validity on such title if the property covered by Archives, and the fact that the proceeding of
the title or patent is part of the public forest. the registration was brought under the Torrens
act which presupposes an existing title to be
confirmed, are all evidences that the land is a
REPUBLIC VS SAYO private land.
(191 SCRA 71)
FACTS: ISSUE: W/N the respondent's evidences can
The case at bar started at 1961 whe be considered as proof that the lot 7454 is a
the spouses Casiano and Luz Sandoval filed private land.
an application for a parcel of land, Lot 7454
originally party of Santiago but had since then HELD:
been transferred to Nueva Vizcaya. No.

Under the Regalian Doctrine 2 all lands


not otherwise appearing to be clearly within

MARIA ANNY G. YANONG JMC COLLEGE OF LAW- LLB 5YR PROGRAM Page 1
NATRES CASE DIGEST

private ownership are presumed to belong to As to the informacion posesoria invoked by the
the State. Hence it is that all applicants in land private respondents, it should be pointed out
registration proceedings have the burden of that under the Spanish Mortgage Law, it was
overcoming the presumption that the land thus considered a mode of acquiring title to public
sought to be registered forms part of the public lands, subject to two (2) conditions: first, the
domain. Unless the applicant succeeds in inscription thereof in the Registry of Property,
showing by clear and convincing evidence that and second, actual, public, adverse, and
the property involved was acquired by him or uninterrupted possession of the land for twenty
his ancestors either by composition title from (20) years (later reduced to ten [10] years); but
the Spanish Government or by possessory where, as here, proof of fulfillment of these
information title, or any other means for the conditions is absent, the informacion posesoria
proper acquisition of public lands, the property cannot be considered as anything more than
must be held to be part of the public domain . 4 prima facie evidence of possession. 7
The applicant must present competent and
persuasive proof to substantiate his claim; he Finally, it was error to disregard the Solicitor
may not rely on general statements, or mere General in the execution of the compromise
conclusions of law other than factual evidence agreement and its submission to the Court for
of possession and title. 5 approval. It is, after all, the Solicitor General,
who is the principal counsel of the
In the proceeding at bar, it appears that Government; this is the reason for our holding
the principal document relied upon and that "Court orders and decisions sent to the
presented by the applicants for registration, to fiscal, acting as agent of the Solicitor General
prove the private character of the large tract of in land registration cases, are not binding until
land subject of their application, was a they are actually received by the Solicitor
photocopy of a certification of the National General."
Library. But, as this Court has already had
occasion to rule, that Spanish document
cannot be considered a title to property, it not SEVILLE VS. NATIONAL
being one of the grants made during the DEVELOPMENT COMPANY
Spanish regime, and obviously not constituting GR NO. 129401
primary evidence of ownership. 6 It is an FEBRUARY 2, 2001
inefficacious document on which to base any FACTS:
finding of the private character of the land in LSBDA (Leyte Sub-A Basin
question. Development Authority) was created by virtue
of PD no. 625. In connection with this, a letter
It thus appears that the decision of the of instruction was executed authorizing LSBDA
Registration Court a quo is based solely on the to purchase or acquire privately owned land.
compromise agreement of the parties. But that
compromise agreement included private In June 14, 1980, certain Calixto Yap
persons who had not adduced any competent sold to LSBDA a 464,920 square meter parcel
evidence of their ownership over the land of land. The LSBDA was able to register it
subject of the registration proceeding. Portions under its name through the approved
of the land in controversy were assigned to miscellaneous sales application with the
persons or entities who had presented nothing Bureau of Lands.
whatever to prove their ownership of any part
of the land. The assent of the Directors of However in May 11, 1990, petitioner
Lands and Forest Development to the Seville filed a complaint for recovery or real
compromise agreement did not and could not property, rentals and damages against LSBDA,
supply the absence of evidence of title required contending that he acquired the property and
of the private respondents. its predecessor-in-interest for more than 30

MARIA ANNY G. YANONG JMC COLLEGE OF LAW- LLB 5YR PROGRAM Page 2
NATRES CASE DIGEST

years and therefore they are the rightful owner Act (Act No. 496). The Republic of the
of it. Philippines, thru the Director of Lands opposed
the application alleging that Pacific Farms, Inc.
The RTC rendered judgment in favor of does not possess a fee simple title to the land
the petitioner, when the case reached in the nor did its predecessors possess the land for at
CA, the latter reversed the RTC decision on least thirty (30) years immediately preceding
the ground that petitioner’s possession of the the filing of application. The opposition likewise
said property was never ripen into ownership specifically alleged that the applicant is a
since it still remains under the inalienable land private corporation disqualified under the
of public domain. (1973) new Philippine Constitution from
acquiring alienable lands of the public domain
ISSUE: citing Section 11, Article 14.
1. WON LSBDA ‘s title to property valid?
2. WON petitioner’s claim of ownership The Director of Forest Development
over the property was correct? also entered its opposition alleging that the
land is within the unclassified public land and,
HELD: hence, inalienable. Other private parties also
The SC ruled in favor of LSBDA. filed their oppositions, but were subsequently
According to the SC, Seville or the Ortega withdrawn. Pacific Farms, Inc. filed a
estate did not become the owner of the subject manifestation-motion to change the applicant
property even though they possessed the land from Pacific Farms, Inc. to J. Antonio Araneta.
for more than 30 years, because at the time
they occupied the said land, it is still part of ISSUE: WON the land known as the "Tambac
inalienable land of public domain. Until the Island" can be subject of registration.
State declares it to be alienable and
disposable, no matter how long they HELD:
possessed it, it may not ripen into ownership. No.

Here the petitioner failed to discharge The Court held that lands of the public
the burden of proving in court that the subject domain are classified under three main
property is already outside the component of categories, namely: Mineral, Forest and
inalienable land of public domain when they Disposable or Alienable Lands. Under the
occupied the same. Commonwealth Constitution, only agricultural
lands were allowed to be alienated. Their
LDBDA is a rightful owner of the said disposition was provided for under
land because, it acquired ownership and title Commonwealth Act No. 141 (Secs. 6-7), which
over it through Sales patent application which states that it is only the President, upon the
was approved by the Bureau of Land, this recommendation of the proper department
shows that the said property is no longer under head, who has the authority to classify the
the ambit of inalienable public land. lands of the public domain into alienable or
disposable, timber and mineral lands. Mineral
and Timber or forest lands are not subject to
private ownership unless they are first
DIRECTOR OF LANDS VS IAC reclassified as agricultural lands and so
219 SCRA 108 released for alienation. In the absence of such
FACTS: classification, the land remains as unclassified
The land involved is an island known as land until released therefrom and rendered
Tambac Island in Lingayen Gulf situated in the open to disposition. Courts have no authority to
Municipality of Bani, Pangasinan. Pacific do so.
Farms, Inc. initially applied for registration the
subject property under the Land Registration

MARIA ANNY G. YANONG JMC COLLEGE OF LAW- LLB 5YR PROGRAM Page 3
NATRES CASE DIGEST

This is in consonance with the Regalian


doctrine that all lands of the public domain Petitioner's main contention was that the land
belong to the State, and that the State is the in question was no longer within the
source of any asserted right to ownership in unclassified public forest land because by the
land and charged with the conservation of such approval of his application for Free Patent by
patrimony. Under the Regalian Doctrine, all the Bureau of Lands, the land was already
lands not otherwise appearing to be clearly alienable and disposable public agricultural
within private ownership are presumed to land. He also claimed that the land was a small
belong to the State. Hence, a positive act of portion of Lot 5139, an area which had been
the government is needed to declassify a forest declared disposable public land by the
land into alienable or disposable land for cadastral court.
agricultural or other purposes.
ISSUE:
The burden of proof in overcoming the
presumption of state ownership of the lands of Whether or not the land is alienable and
the public domain is on the person applying for disposable public land
registration that the land subject of the
application is alienable or disposable. HELD:

Since the subject property is still Under the Regalian Doctrine, all lands not
unclassified, whatever possession the otherwise clearly appearing to be privately-
applicant may have had and however long, owned are presumed to belong to the State.
cannot ripen into private ownership. The Forest lands, like mineral or timber lands which
conversion of subject property does not are public lands, are not subject to private
automatically render the property as alienable ownership unless they under the Constitution
and disposable. become private properties. In the absence of
such classification, the land remains
5 United Paracale vs Dela Rosa (221 unclassified public land until released
SCRA 108) PASAOL, Ric Jason Patlingrao therefrom and rendered open to disposition.

The task of administering and disposing lands


of the public domain belongs to the Director of
6 Republic vs Register of Deeds of Lands, and ultimately the Secretary of
Quezon (224 SCRA 537) PAMISARAN, Agriculture and Natural Resources.
Excel Joy G. Classification of public lands is, thus, an
exclusive prerogative of the Executive
FACTS: Department, through the Office of the
President. Courts have no authority to do so.
Petitioner was awarded a 17-hectare parcel of
land, by virtue of which he was issued an OCT. Thus, in controversies involving the disposition
of public agricultural lands, the burden of
Through an investigation conducted by the overcoming the presumption of state
Bureau of Lands, it was found that the free ownership of lands of the public domain lies
patent acquired by Petitioner was fraudulent. A upon the private claimant.
case for falsification of public documents was
filed by Petitioner was acquitted of the crime. In the present case, Petitioner failed to present
clear, positive and absolute evidence to
Subsequently, the Solicitor-General filed a overcome said presumption and to support his
complaint against Petitioner, praying for the claim.
declaration of nullity of the Free Patent and the
OCT.

MARIA ANNY G. YANONG JMC COLLEGE OF LAW- LLB 5YR PROGRAM Page 4
NATRES CASE DIGEST

Issue: WON the Court of Appeals committed


Moreover, the fact the Petitioner acquired a an error in reversing the decision of the trial
title to the land is of no moment, court and dismissing petitioner’s appeal.
notwithstanding the indefeasibility of title
issued under the Torrens System. The Ruling: NO.
indefeasibility of a certificate of title cannot be Petitioner argued that the Court of Appeals
invoked by one who procured the same by erred in setting aside the trial court's decision
means of fraud. in his favor and dismissing the complaint
because when the Director of Lands allowed
petitioner to file a public land application for
7 Ituralde vs Falcasantos (GR No. said property, it was equivalent to a declaration
128017, January 20, 1999) NALLA, Glene that said land was no longer part
Alacayde
Facts:
On October 17, 1986, Ramon Ituralde, herein of the public domain.
petitioner, acquired by purchase from the heirs We deny the petition. The Court of Appeals
of Pedro Mana-ay a parcel of land, with an correctly held that "the evidence is unrebutted
area of 6.0000 hectares, more or less. that the subject land is within the Forest
However, on November 3, 1986, respondent Reserve Area as per L.C. Map No. 1557
Alfredo Falcasantos applied with the Bureau of certified on August 13, 1951'." and, hence, not
Lands for the award to him of the same parcel capable of private appropriation and
of land under free patent, which application occupation.
was protested by Ramon Ituralde. In Republic vs. Register of Deeds of Quezon,
we held that "Forest land, like mineral timber
On February 7, 1989, the Regional Director of lands which are public lands, are not subject to
Lands rendered a decision giving respondent a private ownership unless they under the
period of one hundred twenty (120) days to Constitution, become private properties. In the
exercise the right to repurchase the land by absence of such classification, the land
reimbursing petitioner of all expenses he remains unclassified public land until released
incurred in the purchase of the property in therefrom and rendered open to disposition."
question, and held in abeyance respondent's In Sunbeam Convenience Foods Inc. vs. Court
application for free patent. Alfredo failed to of Appeals, we said: "Thus, before any land
exercise the right of repurchase, which may be
amounted to a waiver. The Director of Land
then rejected Alfredo’s application and allowed declassified from the forest group and
Ramon Ituralde to file a public land application converted into alienable or disposable land for
for the subject land. Meanwhile, in a case for agricultural or other purposes, there must be a
the recovery of ownership and possession filed positive act from the government. Even rules
by Ituralde against Alfredo, the trial court on the confirmation of imperfect titles do not
rendered a decision declaring petitioner the apply unless and until the land classified as
owner and the possessor of the subject parcel forest land is released in an official
of land with all the improvements existing proclamation to that effect so that it may form
thereon, with an area of 3.1248 hectares, and part of the disposable agricultural lands of the
ordering respondent to vacate the land in public domain."
question. Ramon appealed to the decision. Hence, a positive act of the government is
However the Court of Appeals reversed the needed to declassify a forest land into
decision of the trial court and dismissed alienable or disposable land for agricultural or
Roman’s appeal. Hence this case. other purposes. "
And the rule is "Possession of forest lands,
however long, cannot ripen into private
ownership."

MARIA ANNY G. YANONG JMC COLLEGE OF LAW- LLB 5YR PROGRAM Page 5
NATRES CASE DIGEST

What is more, there is yet no award or grant to The Lower Court denied the application, while
petitioner of the land in question by free patent the CA reversed the decision, affirming the
or other ways of acquisition of public land. surface rights of the Dela Rosas, while also the
Consequently, he cannot lawfully claim to be sub-surface rights of the BCI and Atok. In other
the owner of the land in question. words, the CA ruled that the surface was to be
agricultural and mineral underneath.

8 Republic vs. Court of Appeals and dela ISSUE:


Rosa (GR No. L-43938, 04/15/1988) 1. Who has better claim on the land in
MISTERIO, John Kessler Sumauang dispute
2. WON land can be classified as both
FACTS: agricultural and mineral at the same time.

The case is about a parcel of land whose HELD:


ownership is being disputed by four parties: the
Dela Rosas, Benguet Consolidated Inc. (BCI), 1ST ISSUE:
Atok Corp. (Atok) and the Bureau of Forestry
Development (BFD). The Court held that BCI and Atok have vested
rights over the land. The Court found that the
Sometime in 1965, Jose dela Rosa, on his own mineral claims of both BCI and Atok have been
behalf and on behalf of this three children, filed perfected prior to the approval of the 1935
an application for the registration of a parcel of Constitution. As a legal effect of a valid mineral
land divided into 9 lots in Benguet. According claim, it segregates the area from the public
to the children, they had acquired the land by domain and confers to the locator the
virtue of prescription. As evidenced, they beneficial ownership of the claim. As of 1935,
produced tax declarations and realty tax they were removed from the public domain and
receipts. had become private properties of BCI and
Atok. Even if the land was included in the
BCI and Atok opposed their application Central Cordillera forest reserve, it did not
claiming that the lots were covered by mineral impair the rights vested in both mining
claim sold to it in 1934. BCI, on lots 1-5, companies. The claim of the Dela Rosas was
contented that it had been in actual, continuous disregarded for weak evidence, and even so,
and exclusive possession of the land in the they could not have acquired the land through
concept of an owner. As evidence, BCI prescription since the lad had already been
presented geological maps, payments of taxes converted to mineral land.
and construction on the land.
2ND ISSUE:
Atok claims of ownership on a portion of lots 1-
5 and all of lots 6-9 is similar to that of BCI, that No. Land cannot be classified as half
a mineral claim covering that the lots had been agricultural and half mineral. The Court held
sold to them in 1931 and produced evidence that the classification of land must be
such as the construction and tax payments. categorical: the land must either completely be
mineral or agricultural. In this case, while the
BFD objected that the land in dispute was land was initially classified as forest land, it
covered by the Central Cordillera Forest ceases to be so and completely became
Reserve under Proc. No. 217. And since it mineral when the mining claims were
was a forest land, it cannot be subject to perfected. Even if the surface was being tilled
alienation, pursuant to the 1935 and 1973 by the Dela Rosas, it is still to be considered as
Constitution. mineral land.

MARIA ANNY G. YANONG JMC COLLEGE OF LAW- LLB 5YR PROGRAM Page 6
NATRES CASE DIGEST

As long as mining operations were being petitioners, caused the extrajudicial partition of
undertaken thereon or underneath, it is still the property. Thus, the Register of Deeds
mineral, and not agricultural. issued a TCT under the names of the
heirs/petitioners in 1954.
Hence, the Court ruled that BCI and Atok have
exclusive rights to the property in question by When Concepcion caused the survey and the
virtue of their respective mining claims which subdivision, it was at this point that respondent
they validly acquired before the 1935 brought to the attention of Concepcion that
Constitution which prohibits the alienation of 5,000 sq.m. portion of the property is already
the lands of the public domain except owned by it
agricultural lands. It could not have been based on the deed of donation. Upon learning
transferred to private respondents by virtue of from the ROD that the donation was not
acquisitive prescription not could its use be annotated in the title, petitioners refused to
shared simultaneously by them and the mining recognize the donation. After Concepcion’s
companies for agricultural and mineral death, the heirs/petitioners continued to pursue
purposes. their claims to recover the subject property
from respondent.
Wherefore, the decision of the CA was set
aside, and reinstated the decision of the lower In 2000 or after 6 decades after the alleged
court. deed of donation was executed, petitioners
filed an action for declaration of nullity of deed
of donation and recovery of possession and
9 Heirs of Gozo v. Philippine Union ownership against respondent. Petitioners
Mission Corp. of the Seventh Day Adventist argued that the signatures of their parents in
Church, G.R. No. 195990, [August 5, 2015]) the deed is not genuine and the donation lack
LOZADA, Leah Amaya the formality of acceptance, hence invalid.

FACTS: Respondent argued that since 1937 they are in


open, continuous, exclusive and adverse
Petitioners are heirs of Spouses Rafael and possession of the property in the concept of an
Concepcion Gozo who possessed a property in owner, that they have built therein a church
Brgy. Lala, Kolambugan, Lanao del Norte of and an elementary school and that the
236,638 sq.m. petitioners’ claim of lack of acceptance 63
years after the deed was executed is already
Respondent (PUMC-SDA) claimed that a barred by laches.
portion of that property was donated to it by
spouses Gozo based on the deed of donation In 2004, the RTC rendered a decision in favor
dated 28 Feb. 1937. of petitioners declaring that since the deed is
void for lack of acceptance, they are the rightful
When spouses Gozo donated the property in owners thereof. The Court of Appeals
1937, they were not yet the registered owners reversed the RTC decision on the ground of
of the property although they are the lawful laches stating that petitioners failed to assert
possessors thereof. their rights over the land for

In 1953, the land was registered under the more than 60 years, thus laches had set in.
name of spouses Gozo and an OCT was
issued in their favor pursuant to Homestead ISSUE:
Patent granted by the President of the Phil. Whether or not the donor is necessarily the
owner of the real estate donated?
After Rafael Gozo’s dealth, his wife
Concepcion and their 6 children, herein RULING:

MARIA ANNY G. YANONG JMC COLLEGE OF LAW- LLB 5YR PROGRAM Page 7
NATRES CASE DIGEST

State College cease to exist or be abolished or


No. The donors may be the possessor at the should such parcels of land aforementioned be
time of the donation in 1937, but they are not no longer needed by the State College, the
the owners thereof as the subject property was
same shall revert to the Province of Bataan.
part of the public domain. It was only in 1953
that ownership of the property was vested by On the basis of the above provision, Cong.
the State to the spouses Gozo by virtue of its Garcia wrote to then Governor of Bataan
issuance of the OCT pursuant to the Leonardo Roman, and the Sangguniang
Homestead Patent granted by the President of Panlalawigan of Bataan (petitioner), requesting
the Phil. Hence the donor is not necessarily them to cause the transfer of the title of the
the owner of the real property they donated aforesaid lots to BPSC. No transfer was
when the said property was still part of the
effected. Thus, Cong. Garcia, along with the
public domain (inalienable). The donors cannot
give what they do not have, nemo dat quod faculty members and some concerned
non habet. students of BPSC (collectively, the
respondents) filed a Special Civil Action for
Mandamus with the RTC of Balanga, Bataan
SANGUNIANG PANLALAWIGAN OF against the Governor and the petitioner.
BATAAN VS. GARCIA Initially, the Board of Trustees of the BPSC
G.R. NO. 174964
was impleaded as an unwilling plaintiff but was
OCTOBER 5, 2016
FACTS: eventually included as co-petitioner in the civil
Lot Nos. 2193 and 2194 of the Bataan suit pursuant to Resolution No
Cadastre, containing 1,222 square meters and
10,598 sq m, respectively, were registered in ISSUE: Whether or not the subject parcels of
the name of the Province of Bataan. Both lots land are patrimonial properties of the Province
were embraced in Original Certificate of Title of Bataan which cannot be taken without due
(OCT) No. N-182, and occupied by the Bataan process of law and without just compensation?
Community Colleges (BCC) and the Medina
Lacson de Leon School of Arts and Trades HELD:
(MLLSAT), both State-run schools. On NO. Under the well-entrenched and
February 26, 1998, the Congress of the time-honored Regalian Doctrine, all lands of
Philippines passed Republic Act (R.A.) No. the public domain are under the absolute
8562, authored by Congressman Enrique T. control and ownership of the State. The State's
Garcia, Jr. (Cong. Garcia), converting the ownership of and control over all lands and
MLLSAT into a polytechnic college, to be resources of the public domain are beyond
known as the Bataan Polytechnic State dispute. Section 2, Article XII of the 1987
College (BPSC), and integrating thereto the Constitution provides that “All lands of the
BCC. public domain, waters, minerals, coal,
petroleum and other mineral oils, all forces of
All parcels of land belonging to the potential energy, fisheries, forests or timber,
government occupied by the Medina Lacson wildlife, flora and fauna, and other natural
de Leon School of Arts and Trades and the resources are owned by the State
Bataan Community Colleges are hereby
declared to be the property of the Bataan
Polytechnic State College and shall be titled
under that name: Provided, That should the

MARIA ANNY G. YANONG JMC COLLEGE OF LAW- LLB 5YR PROGRAM Page 8
NATRES CASE DIGEST

time of the grant of the cadastral decree and


REPUBLIC VS. ESPINOSA issuance of title to Espinosa. In land
G.R. NO. 186603 registration proceedings, the applicant has the
APRIL 5, 2017 burden of overcoming the presumption of State
FACTS: ownership. It must establish, through
A cadastral decree was issued in favor incontrovertible evidence, that the land sought
of Espinosa. The Original Certificate of Title to be registered is alienable or disposable
was issued in the name of Espinosa who later based on a positive act of the government.
sold to Caliston which a Transfer Certificate of Since cadastral proceedings are governed by
Title was issued. The State through Regional the usual rules of practice, procedure, and
Executive Director of the DENR filed a evidence, a cadastral decree and a certificate
Complaint for annulment of title and reversion of title are issued only after the applicant
of land with the RTC claiming the property is proves all the requisite jurisdictional facts-that
inalienable public land because it fell within the they are entitled to the claimed lot, that all
timberland area. parties are heard, and that evidence is
considered. As such, the cadastral decree is a
RTC ruled in favor of the State and judgment which adjudicates ownership after
ordered reversion of the property. proving these jurisdictional facts. Here, it is
undisputed that Espinosa was granted a
CA ruled in favor of Espinosa and found cadastral decree and was subsequently issued
that the State failed to prove fraud or OCT No. 191- N, the predecessor title of
misrepresentation when she was issued the Caliston’s TCT No. 91117. Having been
Original Certificate of Title. It further ruled that granted a decree in a cadastral proceeding,
the State failed to prove that the property is Espinosa can be presumed to have overcome
forest land. The lone piece of evidence the presumption that the land sought to be
consisting of LC Map No, 2978 was not registered forms part of the public domain. 33
authenticated pursuant to Section 24 Rule 132 This means that Espinosa, as the applicant,
of the Rules of Court. was able to prove by incontrovertible evidence
that the property is alienable and disposable
It noted that the parties stipulated only property in the cadastral proceedings. This is
as to the existence of the map, but not as to not to say, however, that the State has no
genuineness of truthfulness of its content. remedy to recover the property if indeed it is
Assuming that the map is admitted in evidence, part of the inalienable lands of the public
Espinosa’s rights over the property, which domain. The State may still do so through an
accrued in 1962, should not be prejudiced by a action for reversion, as in the present case.
subsequent classification by the State done in
1986, or after 24 years.
IMPERIUM AND DOMINIUM

ISSUE: Whether or not the State has LEE HONG KOK VS. DAVID
sufficiently proved that the property is part of GR NO.G.R. NO. L-3038
inalienable forest land at the time Espinosa DECEMBER 27, 1972
was granted the cadastral decree and issued a
title. Nature:
Lee Hong Kok, et al filed an appeal on
certiorari seeking to reverse the decision of the
HELD: Court of Appeals which affirmed the decision of
No. the lower court in dismissing the complaint to
have the Torrens Title of Aniano David be
The State failed to prove that the declared null and void.
property was classified as forest land at the

MARIA ANNY G. YANONG JMC COLLEGE OF LAW- LLB 5YR PROGRAM Page 9
NATRES CASE DIGEST

FACTS: There is no legal justification for


Aniano David acquired lawful title to a nullifying the right to the disputed lot arising
parcel of land pursuant to his miscellaneous from the grant made in his favor.
sales application. An order of award and for
issuance of a sales patent was made by the 1. The lot in question is NOT a private
Director of Lands on June 18, 1958, covering property as the Director of Lands and
Lot 2892 containing an area of 226 square the Secretary of Agriculture and Natural
meters, which is a portion of Lot 2863 of the Resources have always sustained its
Naga Cadastre. public character for having been formed
by reclamation and not accretion.
On the basis of the order of award of the
Director of Lands, the Undersecretary of a. RECLAMATION is the act of
Agriculture and Natural Resources issued filling submerged land by
Miscellaneous Sales Patent No. V-1209 deliberate acts and reclaiming
pursuant to which an OCT was issued by the title thereto.
Register of Deeds of Naga City to Aniano
David. b. ACCRETION - is the process
whereby the soil is deposited;
Since the filing of the sales application of accretion resulting from the
Aniano David and during all the proceedings in gradual deposit by or
connection with said application, up to the sedimentation from the waters
actual issuance of the sales patent in his favor, belongs to the owners of the
Lee Hong Kok did not put up any land bordering on streams,
opposition or adverse claim to Lot 2892. torrents, lakes, or rivers

The opposition was fatal because after the i. Therefore the only
registration and issuance of the certificate and remedy available to the
duplicate certificate of title based on a public appellants is an action
land patent, the land covered thereby for reconveyance on the
automatically comes under the operation of RA ground of fraud.
496 subject to all the safeguards provided
therein. Under Section 38 of RA 496, any QUESTION:
question concerning the validity of the But did he commit any fraud?
certificate of title based on fraud should be
raised within 1 year from the date of the NO. Aniano David has not
issuance of the patent otherwise the certificate committed any fraud in applying for the
of title becomes indefeasible after the lapse of purchase of the Lot because everything
1 year. was done in the open. The notices
regarding the auction sale of the land
The contention of Lee Hong Kok was that were published, the actual sale and
David’s lot is a private property for it was award thereof to Aniano David were not
formed thru the process of accretion. clandestine but open and public official
acts of an officer of the Government.
The application was merely a renewal of
ISSUE 1: Can the patent certificate of Aniano his deceased wife's application who had
David to the property be nullified? occupied the land since 1938.

QUESTION: If the grant was presumed to be


HELD: invalid, who has the right to question it?
No.

MARIA ANNY G. YANONG JMC COLLEGE OF LAW- LLB 5YR PROGRAM Page 10
NATRES CASE DIGEST

c. Only the Government, g. The manifestation of the concept


represented by the Director of of jura regalia, which was
Lands or the Secretary of adopted by the present
Agriculture and Natural Constitution, was embodied in
Resources can bring an action the universal feudal theory that
to cancel a void certificate of all lands were held from the
title pursuant to a void patent. Crown, the ownership however
is vested in the state rather
d. The private parties cannot than the head thereof.
claim that the patent and title
issued for the land involved are h. As to the unappropriated public
void since they are not the lands constituting the public
registered owners thereof nor domain, the sole power of
had they been declared as legislation is vested in Congress.
owners in the cadastral
proceedings of the Naga i. There being no evidence
Cadastre after claiming it as their whatever that the property in
private property. question was ever acquired by
the applicants or their ancestors
e. Citing Maninang vs Consolacion: either by composition title from
‘The fact that the grant was made the Spanish Government or by
by the government is undisputed. possessory information title or by
Whether the grant was in any other means for the
conformity with the law or not is a acquisition of public lands, the
question which the government property must be held to be
may raise, but until it is raised by public domain (Heirs of Datu
the government and set aside, Pendatun v. Director of Lands).
the defendant cannot question it.
The legality of the grant is a j. “No public land can be
question between the grantee acquired by private persons
and the government.’ without any grant, express or
implied, from the government.”
By what authority does the government Therefore it is indispensable that
have in disposing of the land in question? there be a showing of a title from
the state or any other mode of
f. IMPERIUM refers to the acquisition recognized by law
government authority possessed otherwise the property is and
by the state which is remains part of the public
appropriately embraced in the domain.
concept of sovereignty.
2. Was David’s title to the property already
DOMINIUM, on the other hand, is indefeasible?
appropriate with reference to
lands held by the state in its a. As far back as 1919, in Aquino
proprietary character. In such v. Director of Lands, Justice
capacity, it may provide for the Malcolm, speaking for the Court,
exploitation and use of lands and stated: "The proceedings under
other natural resources, including the Land Registration Law and
their disposition, except as limited under the provisions of
by the Constitution. Chapter VI of the Public Land
Law are the same in that both

MARIA ANNY G. YANONG JMC COLLEGE OF LAW- LLB 5YR PROGRAM Page 11
NATRES CASE DIGEST

are against the whole world, IPRA LAW (R.A. 8731)


both take the nature of judicial CRUZ VS SECRETARY OF DENR
proceedings, and for both the GR NO. 135385
decree of registration issued is DEC. 6, 2000
conclusive and final." FACTS:
Petitioners Isagani Cruz and Cesar
b. Meanwhile in Cabacug v. Lao. Europa filed a suit for prohibition and
There is this revealing excerpt mandamus as citizens and taxpayers, assailing
appearing in that decision: "It is the constitutionality of certain provisions of
said, and with reason, that a Republic Act No. 8371, otherwise known as the
holder of a land acquired under Indigenous People’s Rights Act of 1997 (IPRA)
a free patent is more favorably and its implementing rules and regulations
situated than that of an owner (IRR). The petitioners assail certain provisions
of registered property. Not only of the IPRA and its IRR on the ground that
does a free patent have a force these amount to an unlawful deprivation of the
and effect of a Torrens Title, but State’s ownership over lands of the public
in addition the person to whom it domain as well as minerals and other natural
is granted has likewise in his resources therein, in violation of the regalian
favor the right to repurchase doctrine embodied in section 2, Article XII of
within a period of five years." It is the Constitution.
quite apparent, therefore, that
petitioners' stand is legally ISSUE:
indefensible. Do the provisions of IPRA contravene the
Constitution?
RULING:
The decision of the CA of January 31, HELD:
1969 and its resolution of March 14, No, the provisions of IPRA do not
1969 are affirmed. contravene the Constitution. Examining the
IPRA, there is nothing in the law that grants to
IMPORTANT CONCEPTS the ICCs/IPs ownership over the natural
resources within their ancestral domain.
IMPERIUM DOMINIUM Ownership over the natural resources in the
The State’s authority to govern is 1. The capacity of the State ancestral domains remains with the State and
embraced in the concept of to own and acquire the rights granted by the IPRA to the ICCs/IPs
sovereignty that includes passing property. It covers such
laws concerning a territory, rights as title to land, over the natural resources in their ancestral
maintaining peace and order over it, exploitation and use of it, domains merely gives them, as owners and
and defending it against foreign and disposition or sale of
invasion. the same. occupants of the land on which the resources
2. It refers to lands held by are found, the right to the small scale utilization
the government in a
proprietary character: can of these resources, and at the same time, a
provide for the exploitation priority in their large scale development and
and use of lands and
other natural resources. exploitation.
When the State acts in this capacity, The Regalian Doctrine whereby all
jure imperii, it generally enjoys lands of the public domain belong to
Additionally, ancestral lands and ancestral
state immunity. the State, and anyone claiming title domains are not part of the lands of the public
has the burden to show ownership,
comes within this concept. In this
domain. They are private lands and belong to
capacity, jure gestium, the State the ICCs/IPs by native title, which is a concept
descends to the status of ordinary
persons and thus becomes liable as
of private land title that existed irrespective of
such. any royal grant from the State. However, the
right of ownership and possession by the
ICCs/IPs of their ancestral domains is a limited

MARIA ANNY G. YANONG JMC COLLEGE OF LAW- LLB 5YR PROGRAM Page 12
NATRES CASE DIGEST

form of ownership and does not include the accordance with Igorot custom. However, no
right to alienate the same. document of title had been issued from the
Spanish Crown
CARIÑO VS. INSULAR
GOVERNMENT ISSUE: Whether or not Mateo is the rightful
41 PHIL. 935 (1906) owner of the land by virtue of his possession of
it for some time.
FACTS:
Mateo Carino (appellant) filed his HELD:
petition in the Court of Land Registration to be No.
granted a parcel of land consisting of 146
hectares in Baguio, Province of Benguet The statute of limitations did not run
together with a house erected thereon. against the government. The government is
still the absolute owner of the land (regalian
It was granted, but the Government of doctrine). Further, Mateo’s possession of the
the Philippines and also on behalf of the United land has not been of such a character as to
States averred having taken possession of the require the presumption of a grant. No one has
property for public and military purposes, lived upon it for many years. It was never used
for anything but pasturage of animals, except
Respondents also asserted that they insignificant portions thereof, and since the
had title to all the land in the Philippines except insurrection against Spain it has apparently not
to permit private lands to be acquired and no been used by the petitioner for any purpose.
prescription runs against the Spanish crown.
While the State has always recognized
The US succeeded the title of Spain the right of the occupant to a deed if he proves
through Treaty of Paris and Mateo’s land was a possession for a sufficient length of time, yet
not registered and that he had lost all his it has always insisted that he must make that
rights and now is deemed to be a mere proof before the proper administrative officers,
trespasser. and obtain from them his deed, and until he did
the State remained the absolute owner.
Then the Court of First Instance
dismissed the application for Mateo since he
did not possessed the land since time “In view of these provisions of the law, it
immemorial and the land was property of the seems to us impossible to say that as to the
Government. public agricultural lands in the Philippines there
existed a conclusive presumption after a lapse
The decision was affirmed by the of thirty or any other number of years that the
Philippine Supreme Court. Government of Spain had granted to the
possessor thereof a legal title thereto.
Thus the case was brought to the US
Supreme Court by virtue of Writ of The plaintiff is not entitled to the benefits of
Error(general method of bringing cases to this paragraph 6 of section 54 of Act No. 926, the
court, an appeal the exception, confined to Public Land Act, for the reason that the act is
equity in the main. not applicable to the Province of Benguet. The
judgment of the court below is affirmed, with
Mateo Carino in his appeal stated that he is an the costs of this instance against the
Igorot of the Province of Benguet, they have appellant.”
owned the land for more than 50 years before
the Treaty of Paris ,they all had been
recognized as owners by the Igorots and had
inherited or received his land from his father in

MARIA ANNY G. YANONG JMC COLLEGE OF LAW- LLB 5YR PROGRAM Page 13
NATRES CASE DIGEST

ACTING REGISTRARS OF LAND TITLES Registration, later Justice of this Court, had
AND DEEDS OF PASAY CITY VS. RTC stamped his imprimatur.
BRANCH 57 MAKATI
GR NO. 81564 On October 12, 1987, the respondent
APRIL 26, 1990 court issued an order which grants the
plaintiffs' prayer in the OMNIBUS MOTION in
FACTS: order to safeguard the integrity of the land
On November 5, 1985, Domingo embraced in OCT 291 and authorizes Plaintiff
Palomares, as administrator of the heirs of Domingo C. Palomares:
Delfin Casal, commenced suit with the
Regional Trial Court, Branch 132, Makati, 1. To order such subdivision and/or
Metro Manila for declaratory relief, quieting of individual survey or surveys within
title, cancellation of Transfer Certificate of Title Parcel II, Parcel III and Parcel IV under
No. 192, and cancellation of entries upon Survey Plan Psu-2031 by a licensed
Original Certificate of Title No. 291. geodetic engineer or engineers at
plaintiffs' expense in order to facilitate
Palomares had earlier come to this and simplify the efficient administration
Court (February 27, 1985) on a similar petition, of the property described in OCT 291;
and in addition, to direct the Register of Deeds and
to issue a duplicate owner's copy of Original
Certificate of Title No. 291, embracing 2. To sell, exchange, lease or otherwise
allegedly Hacienda de Maricaban, in lieu of the dispose (of) any area or areas or portion
(alleged) lost one. On September 9, 1985, the or portions thereof, subject to the
Court denied the petition for lack of merit. approval of the Intestate Estate Court, to
cover expenses for the payment of
taxes to which the property is subject,
During the pendency of the case also, as well as expenses of administration
Proclamation No. 192 (“RESERVING FOR and for the protection of the integrity of
THE VETERANS CENTER SITE PURPOSES the said lands.
CERTAIN PARCEL OF LAND OF THE
PUBLIC DOMAIN SITUATED IN THE Eleven days later, or on October 23,
PROVINCE OF RIZAL, ISLAND OF LUZON”) 1987, it issued another order acting on the
and Proclamation No. 423 (“RESERVING FOR plaintiffs MOTION dated October 15, 1987
MILITARY PURPOSES CERTAIN PARCELS praying for the issuance of a Writ of Execution
OF THE PUBLIC DOMAIN SITUATED IN THE implementing the Order of the Court dated
MUNICIPALITY OF PASIG, TAGUIG, AND October 12, 1987, to execute and perform the
PARAÑAQUE, PROVINCE OF RIZAL, AND acts authorized in the said Order of October
PASAY CITY”) were issued by the 12, 1987 without the need of a Writ of
government. Execution, he may take whatever steps he
considers appropriate for the implementation of
On August 29, 1986, the respondent the said Order without need of further Orders
judge issued a temporary restraining order, or additional authority from the Court.
directing the petitioners to cease and desist
from performing the acts complained of. The petitioners filed a notice of appeal;
the respondent court, however, denied it" 3 "it
In a subsequent memorandum, the being directed against . . . an interlocutory
petitioners alleged that Dolores Casal had order.
conveyed the property to the Government of
the United States in 1906 and the Manila Hence, this recourse.
Railroad Company on which Judge Ostrand,
the Presiding Judge of the Court of Land

MARIA ANNY G. YANONG JMC COLLEGE OF LAW- LLB 5YR PROGRAM Page 14
NATRES CASE DIGEST

ISSUE: Whether or not, Proclamation 192 and Claims that Judge Ostrand's decree was
423 were valid in the absence of proof of a counterfeit is not only self-serving, it finds no
ownership of the property by the government? support from the records.

HELD: The presumptions is "that official duty


Yes. has been regularly performed," and the burden
is on the private respondent to prove irregular
While there is no presumption that performance. The barren insistence that Judge
property is Government property until Ostrands order was a forgery is not sufficient to
otherwise shown, because the law recognizes overthrow the presumption. To begin with, the
private ownership, thus: act of forgery has been seasonably disputed by
the petitioners. Secondly, the Acting Registrar
Art. 425. Property of private ownership, of Deeds of Pasig, who supposedly certified to
besides the patrimonial property of the the fake character of Judge Ostrand's order,
State, provinces, cities, and has himself joined the other petitioners in
municipalities, consists of all property opposing the reconveyance sought.
belonging to private persons, either
individually or collectively.
CLASSIFICATION OF LANDS
we find hard evidence on record that: A. CONSTITUTION

(1) the property covered by OCT No. PALOMO V. CA


291 had been conveyed to the United 266 SCRA 392
States of America; G.R. NO. 130906

(2) it had been later ceded to the FACTS:


Republic of the Philippines, and In 1913, some 440,530 square meters
of land in Albay were reserved for provincial
(3) as a consequence, OCT No. 291 park purposes by virtue of EO No. 40. Of said
was cancelled upon final order of Judge area, 15 parcels of land were registered in the
Ostrand. name of Diego Palomo by the Court of First
Instance.
Be that as it may, the private respondent
in G.R. No. 81564 is pressed hard to establish In 1937, Diego Palomo donated these
the fact that portions of the property, especially lands to his heirs Ignacio and Carmen Palomo.
the open spaces referred to in the lower court's Claiming that the aforesaid original certificates
writ of injunction and the private respondent's of title were lost during the Japanese
manifestation of December 14, 1989, and occupation, Ignacio Palomo filed a petition for
which open spaces it claims to be reconstitution with the Court of First Instance of
outside Maricaban, are indeed outside Albay on May 30, 1950. The Register of
Maricaban (or OCT 291). Deeds of Albay issued Transfer Certificates of
Title Nos. 3911, 3912, 3913 and 3914
With respect, however, to parts thereof sometime in October 1953.
on which Fort Bonifacio, Libingan ng mga
Bayani, Ninoy Aquino International Airport, On July 10, 1954, President Magsaysay issued
Nayong Pilipino, Population Commission Proclamation No. 47 converting the area
National Science and Development Board, and covered by EO 40 into the Tiwi Hot Spring
National Housing Authority sit, the hands of the National Park. The Palomos contended that
private respondent are tied. they have been in possession of the subject
lands and have introduced improvements
thereon.

MARIA ANNY G. YANONG JMC COLLEGE OF LAW- LLB 5YR PROGRAM Page 15
NATRES CASE DIGEST

CENRO of Antipolo City declared the land to


ISSUE: Were the Original Certificate of Titles fall within the alienable and disposable zone.
issued to the petitioners valid?
HELD: On the other hand, the Land
NO. Registration Authority recommended the
exclusion of lot no. 3138-B on the ground that it
Before the Treaty of Paris in 1899, the is a legal easement and intended for public
lands, whether agricultural, mineral, or forest use, hence, inalienable and indisposable.
were under the exclusive patrimony and
dominion of the Spanish crown. Private On July 2001, the Republic of the
ownership of land could only be acquired Philippines, the Laguna Lake Development
through royal concessions which were Authority (LLDA) filed its opposition which
documented in various forms, such as (1) alleged that the lot subject of the application for
Titulo Real or Royal Grant," (2) Concession registration may not be alienated and disposed
Especial or Special Grant, (3) Titulo de since it is considered part of the Laguna Lake
Compra or Title by Purchase and (4) Bed, a public land within, its jurisdiction.
Informacion Posesoria or Possessory
Information title obtained under the Spanish ISSUE: WON the property subject of the
Mortgage Law or under the Royal Decree of amended application is alienable and
January 26, 1889. disposable property of the State.

The decision of the CFI relied upon by HELD:


petitioners were not signed by the judge but Yes. The property subject of this
were merely certified copies of notification to application was alienable and disposable
Diego Palomo bearing the signature of the public agricultural land.
Clerk of Court.
However, respondent failed to prove
More importantly, the lands in question were that it possesses registrable title over the
not classified as alienable lands. Since the property. The statute of limitations with regard
lands were made part of a reservation for to public agricultural lands does not operate
provincial park purposes, they form part of the against the statute unless the occupant proves
forest zone. Thus, they cannot be the valid possession and occupation of the same after a
subject of alienation. claim of ownership for the required number of
years to constitute a grant from the State. A
mere casual cultivation of portions of the land
REPUBLIC VS CANDY MAKER INC by the claimant does not constitute sufficient
GR. NO. 163766 basis for a claim of ownership, such
JUNE 22, 2006 possession is not exclusive and notorious as to
give rise to presumptive grant from the state.
FACTS:
On April, 29, 1999, Antonia, Eladia Cruz In light of the foregoing, the petition of
and Felisa Cruz executed a Deed of Absolute the Republic of the Philippines is granted.
Sale in favor of Candy Maker, Inc. for a parcel
of land located below the reglementary lake
elevation of 12.50m, about 900 meters away CHAVEZ VS PUBLIC ESTATES AUTHORITY
the Laguna de Bay. G.R. NO. 133250
JULY 9, 2002
Candy Maker, Inc. as applicant, filed an FACTS:
application with the MTC of Taytay, Rizal for On November 20, 1973, the
registration of its alleged title over the lot. The government, through the Commissioner of
Public Highways, signed a contract with the

MARIA ANNY G. YANONG JMC COLLEGE OF LAW- LLB 5YR PROGRAM Page 16
NATRES CASE DIGEST

Construction and Development Corporation of areas of land reclaimed by CDCP in the


the Philippines (“CDCP” for brevity) to reclaim MCCRRP as of December 30, 1981
certain foreshore and offshore areas of Manila which have not yet been sold,
Bay. The contract also included the transferred or otherwise disposed of by
construction of Phases I and II of the Manila- CDCP as of said date, which areas
Cavite Coastal Road. CDCP obligated itself to consist of approximately Ninety-Nine
carry out all the works in consideration of fifty Thousand Four Hundred Seventy Three
percent of the total reclaimed land. (99,473) square meters in the Financial
Center Area covered by land pledge No.
On February 4, 1977, then President 5 and approximately Three Million Three
Ferdinand E. Marcos issued Presidential Hundred Eighty Two Thousand Eight
Decree No. 1084 creating PEA. PD No. 1084 Hundred Eighty Eight (3,382,888)
tasked PEA “to reclaim land, including square meters of reclaimed areas at
foreshore and submerged areas,” and “to varying elevations above Mean Low
develop, improve, acquire, x x x lease and sell Water Level located outside the
any and all kinds of lands.” Financial Center Area and the First
Neighborhood Unit.”
On the same date, then President
Marcos issued Presidential Decree No. 1085 On January 19, 1988, then President
transferring to PEA the “lands reclaimed in the Corazon C. Aquino issued Special Patent No.
foreshore and offshore of the Manila 3517, granting and transferring to PEA “the
Bay”2 under the Manila-Cavite Coastal Road parcels of land so reclaimed under the Manila-
and Reclamation Project (MCCRRP). Cavite Coastal Road and Reclamation Project
(MCCRRP) containing a total area of one
On December 29, 1981, then President million nine hundred fifteen thousand eight
Marcos issued a memorandum directing PEA hundred ninety four (1,915,894) square
to amend its contract with CDCP, so that “[A]ll meters.” Subsequently, on April 9, 1988, the
future works in MCCRRP x x x shall be funded Register of Deeds of the Municipality of
and owned by PEA.” Accordingly, PEA and Parañaque issued Transfer Certificates of Title
CDCP executed a Memorandum of Agreement Nos. 7309, 7311, and 7312, in the name of
dated December 29, 1981, which stated: PEA, covering the three reclaimed islands
known as the “Freedom Islands” located at the
“(i) CDCP shall undertake all southern portion of the Manila-Cavite Coastal
reclamation, construction, and such Road, Parañaque City. The Freedom Islands
other works in the MCCRRP as may be have a total land area of One Million Five
agreed upon by the parties, to be paid Hundred Seventy Eight Thousand Four
according to progress of works on a unit Hundred and Forty One (1,578,441) square
price/lump sum basis for items of work meters or 157.841 hectares.
to be agreed upon, subject to price
escalation, retention and other terms On April 25, 1995, PEA entered into a
and conditions provided for in Joint Venture Agreement (“JVA” for brevity)
Presidential Decree No. 1594. All the with AMARI, a private corporation, to develop
financing required for such works shall the Freedom Islands. The JVA also required
be provided by PEA. the reclamation of an additional 250 hectares
xxx of submerged areas surrounding these islands
to complete the configuration in the Master
(iii) x x x CDCP shall give up all its Development Plan of the Southern
development rights and hereby agrees Reclamation Project-MCCRRP. PEA and
to cede and transfer in favor of PEA, all AMARI entered into the JVA through
of the rights, title, interest and negotiation without public bidding.4 On April 28,
participation of CDCP in and to all the 1995, the Board of Directors of PEA, in its

MARIA ANNY G. YANONG JMC COLLEGE OF LAW- LLB 5YR PROGRAM Page 17
NATRES CASE DIGEST

Resolution No. 1245, confirmed the JVA.5On Officer Sergio Cruz composed the negotiating
June 8, 1995, then President Fidel V. Ramos, panel of PEA.
through then Executive Secretary Ruben
Torres, approved the JVA. On April 13, 1998, Antonio M. Zulueta
filed before the Court a Petition for Prohibition
On November 29, 1996, then Senate with Application for the Issuance of a
President Ernesto Maceda delivered a privilege Temporary Restraining Order and Preliminary
speech in the Senate and denounced the JVA Injunction docketed as G.R. No. 132994
as the “grandmother of all scams.” As a result, seeking to nullify the JVA. The Court dismissed
the Senate Committee on Government the petition “for unwarranted disregard of
Corporations and Public Enterprises, and the judicial hierarchy, without prejudice to the
Committee on Accountability of Public Officers refiling of the case before the proper court.”12
and Investigations, conducted a joint On April 27, 1998, petitioner Frank I. Chavez
investigation. The Senate Committees reported (“Petitioner” for brevity) as a taxpayer, filed the
the results of their investigation in Senate instant Petition for Mandamus with Prayer for
Committee Report No. 560 dated September the Issuance of a Writ of Preliminary Injunction
16, 1997.7 Among the conclusions of their and Temporary Restraining Order. Petitioner
report are: contends the government stands to lose
billions of pesos in the sale by PEA of the
(1) the reclaimed lands PEA seeks to reclaimed lands to AMARI. Petitioner prays
transfer to AMARI under the JVA are that PEA publicly disclose the terms of any
lands of the public domain which the renegotiation of the JVA, invoking Section 28,
government has not classified as Article II, and Section 7, Article III, of the 1987
alienable lands and therefore PEA Constitution on the right of the people to
cannot alienate these lands; information on matters of public concern.
Petitioner assails the sale to AMARI of lands of
(2) the certificates of title covering the the public domain as a blatant violation of
Freedom Islands are thus void, and Section 3, Article XII of the 1987 Constitution
prohibiting the sale of alienable lands of the
(3) the JVA itself is illegal. public domain to private corporations. Finally,
petitioner asserts that he seeks to enjoin the
On December 5, 1997, then President loss of billions of pesos in properties of the
Fidel V. Ramos issued Presidential State that are of public dominion.
Administrative Order No. 365 creating a Legal
Task Force to conduct a study on the legality of On December 28, 1998, petitioner filed
the JVA in view of Senate Committee Report an Omnibus Motion:
No. 560. The members of the Legal Task
Force were the Secretary of Justice,8 the Chief (a) to require PEA to submit the terms of
Presidential Legal Counsel,9 and the the renegotiated PEA-AMARI contract;
10
Government Corporate Counsel. The Legal
Task Force upheld the legality of the JVA, (b) for issuance of a temporary
contrary to the conclusions reached by the restraining order; and
Senate Committees.
(c) to set the case for hearing on oral
On April 4 and 5, 1998, the Philippine argument. Petitioner filed a Reiterative
Daily Inquirer and Today published reports that Motion for Issuance of a TRO dated
there were on-going renegotiations between May 26, 1999, which the Court denied in
PEA and AMARI under an order issued by then a Resolution dated June 22, 1999.]
President Fidel V. Ramos. According to these
reports, PEA Director Nestor Kalaw, PEA In a Resolution dated March 23, 1999,
Chairman Arsenio Yulo and retired Navy the Court gave due course to the petition and

MARIA ANNY G. YANONG JMC COLLEGE OF LAW- LLB 5YR PROGRAM Page 18
NATRES CASE DIGEST

required the parties to file their respective alienate. In their present state, the
memoranda. 592.15 hectares of submerged areas
are inalienable and outside the
On March 30, 1999, PEA and AMARI commerce of man.
signed the Amended Joint Venture Agreement
(“Amended JVA,” for brevity). On May 28, 3. Since the Amended JVA seeks to
1999, the Office of the President under the transfer to AMARI, a private corporation,
administration of then President Joseph E. ownership of 77.34 hectares110 of the
Estrada approved the Amended JVA. Freedom Islands, such transfer is void
Due to the approval of the Amended JVA by for being contrary to Section 3, Article
the Office of the President, petitioner now XII of the 1987 Constitution which
prays that on “constitutional and statutory prohibits private corporations from
grounds the renegotiated contract be declared acquiring any kind of alienable land of
null and void.” the public domain.

ISSUE: WON the transfer to AMARI lands 4. Since the Amended JVA also seeks
reclaimed or to be reclaimed as part of the to transfer to AMARI ownership of
stipulations in the (Amended) JVA between 290.156 hectares111 of still submerged
AMARI and PEA violate Sec. 3 Art. XII of the areas of Manila Bay, such transfer is
1987 Constitution. void for being contrary to Section 2,
Article XII of the 1987 Constitution which
HELD: prohibits the alienation of natural
resources other than agricultural lands
On the issue of Amended JVA as of the public domain.
violating the constitution:
PEA may reclaim these submerged
1. The 157.84 hectares of reclaimed areas. Thereafter, the government can classify
lands comprising the Freedom Islands, the reclaimed lands as alienable or disposable,
now covered by certificates of title in the and further declare them no longer needed for
name of PEA, are alienable lands of the public service. Still, the transfer of such
public domain. PEA may lease these reclaimed alienable lands of the public domain
lands to private corporations but may to AMARI will be void in view of Section 3,
not sell or transfer ownership of these Article XII of the 1987Constitution which
lands to private corporations. PEA may prohibits private corporations from acquiring
only sell these lands to Philippine any kind of alienable land of the public domain.
citizens, subject to the ownership
limitations in the 1987 Constitution and
existing laws.

2. The 592.15 hectares of submerged


areas of Manila Bay remain inalienable
natural resources of the public domain
until classified as alienable or
disposable lands open to disposition
and declared no longer needed for
public service. The government can
make such classification and declaration
only after PEA has reclaimed these
submerged areas. Only then can these
lands qualify as agricultural lands of the
public domain, which are the only
natural resources the government can
MARIA ANNY G. YANONG JMC COLLEGE OF LAW- LLB 5YR PROGRAM Page 19
NATRES CASE DIGEST

17 Alba vs Court of Appeals (GR No. Public lands are classified into (1) alienable or
120066, 9/9/1999) ENERO, Jomari Ivan disposable lands which includes agricultural
Tagud lands and (2) inalienable or non-disposable
lands or those not susceptible of private
GR 120066, September 9, 1999 appropriation which includes Timber lands and
Mineral lands. For purposes of administration
FACTS: and disposition, the lands of the public domain
In 1958, Lachica filed an application for title to classified as "disposable" or "alienable" are
a 4,485 sq.m. parcel of land which he had further sub-classified into a.] Agricultural; b.]
acquired through purchase dating back to Residential, commercial, industrial or for similar
1945. However, some parties filed their productive purposes; c.] Educational,
opposition thereto invoking that they are charitable or other similar purposes, and d.]
owners of some parts of land within the total Reservations for town sites and for public and
area applied for. The lower court ruled in quasi-public purposes.
favor of Lachica and held, among others, that
Lachica had been in actual, open and Possession of public agricultural land, however
continuous possession of the subject land in long the period may have extended, never
the concept of owner since 1945 and thus had confers title thereto upon the possessor and it
acquired it through prescription. is because the statute of limitations with regard
to public agricultural land does not operate
ISSUE: against the State, unless the occupant can
Did Lachica acquire the subject property by prove possession and occupation of the same
prescription? under claim of ownership for the required
number of years to constitute a grant from the
RULING: State.
The Court ruled on the negative.
Under (b), Sec. 48, CA 141, confirmation of an
Based from the facts and evidences presented, imperfect title to a public domain requires that:
it was proven that Lachicha only had a title to a 1. There be an open, continuous, exclusive
620sq.m. portion of the total area. Prescription and notorious possession and occupation of
cannot be had on the remaining area as the agricultural lands of the public domain;
Court held that the law applicable in this case 2. It should be under a bona fide claim of
is Sec. 48 of CA 141 (wc deals with registration ownership; and
of lands of public domain) and not Sec. 19 of 3. possession should be for at least thirty
Act 496 (wc deals with registration of private years immediately preceding the filing of the
lands) and with which the lower courts had application for confirmation of title except when
relied on. prevented by war or force majeure

The law in force at the time an action accrues In this case, Lachica had not yet satisfied the
is what governs the proceeding consistent with requirement of the 30 years possession,
the fundamental dictum that laws shall have no hence, prescription cannot be granted in favor
retroactive effect, unless the contrary is of him. JOM
proved. In this case, the lower courts relied on
the provisions on prescription with the
assumption that the subject property is a 18 Republic vs Imperial (GR No. 130906,
private land. However, the application for 2/11/1990) DALISAY, Armando, Jr. D
registration should be that of a judicial
confirmation of an imperfect title considering Case Digest: Republic vs. Imperial
that the land is presumed under the Regalian
Doctrine to be part of the public domain. FACTS:

MARIA ANNY G. YANONG JMC COLLEGE OF LAW- LLB 5YR PROGRAM Page 20
NATRES CASE DIGEST

On September 12, 1917, the late Elias failure to file the appellants brief within the
Imperial was issued Original Certificate of extended period granted to petitioner.
Title (OCT) 408 (500) pursuant to Decree No.
55173 of then Court of First Instance of Hence, the present petition.
Albay. OCT No. 55173 was subdivided and
further subdivided resulting in the issuance ISSUE: Whether or not the petition should
of several titles, which are now the subjects be granted.
of herein petition in the name of private
respondents. Petitioner Republic of the HELD: Yes.
Philippines filed a case with the trial court At the core of the controversy is whether
to judicially declare the Transfer the parcels of land in question are
Certificates of Title (TCT) issued to herein foreshore lands. Foreshore land is a part of
private respondents null and void on the the alienable land of the public domain and
ground that the subject land, on which the may be disposed of only by lease and not
OCT was based, has the features of a otherwise. It was defined as "that part (of
foreshore land based on an investigation the land) which is between high and low
conducted by the DENR, Region V, Legazpi water and left dry by the flux and reflux of
City. Respondents, on the other hand the tides." It is also known as "a strip of
contend that Director of Lands found Jose land that lies between the high and low
Baritua's land covered by TCT No.18655, water marks and, is alternatively wet and
which stemmed from OCT 408(500), to be dry according to the flow of the tide."
"definitely outside of the foreshore area."
The classification of public lands is a
Within the time for pleading, private function of the executive branch of
respondents EANCRA Corporation, Lolita government, specifically the director of
Alcazar and Salvador Alcazar filed their lands (now the director of the Lands
answer with cross-claim, while the rest, Management Bureau). The decision of the
namely, Felix S. Imperial, Feliza S. Imperial, director of lands when approved by the
Elias S. Imperial and Miriam S. Imperial filed Secretary of the Department of
a motion to dismiss. They contended that Environment and Natural Resources
the adjudication by the cadastral court is (DENR) as to questions of fact is conclusive
binding against the whole world including upon the court.
the Republic since the cadastral
proceedings are in rem and the government There is allegedly a conflict between the
itself through the Director of Lands findings of the Director of Lands and the
instituted the proceedings and was a direct DENR, Region V, in the present case.
and active participant therein. Petitioner, Respondents contend that the Director of
through the Office of the Solicitor General, Lands found Jose Baritua's land covered by
filed an objection to the motion to dismiss. TCT No.18655, which stemmed from OCT
After hearing the motion to dismiss, the 408(500), to be "definitely outside of the
trial court dismissed the complaint on the foreshore area." Petitioner, on the other
ground that the judgment rendered by the hand, claims that subsequent investigation
cadastral court in G.R. Cad. Rec. No. 88 and of the DENR, Region V, Legazpi City,
the Courts resolution in the petition to quiet disclosed that the land covered by OCT No.
title, G.R. 85770, both decreed that the 408 (500) from whence the titles were
parcel of land covered by OCT No. 408 (500) derived "has the features of a foreshore
was not foreshore. Petitioner appealed to land." The contradictory views of the
the Court of Appeals. The appellate court Director of Lands and the DENR, Region V,
denied petitioners motion for Legazpi City, on the true nature of the land,
reconsideration for lack of merit and for which contradiction was neither discussed
nor resolved by the RTC, cannot be the

MARIA ANNY G. YANONG JMC COLLEGE OF LAW- LLB 5YR PROGRAM Page 21
NATRES CASE DIGEST

premise of any conclusive classification of and their predecessors- in-interest have been
the land involved. in open, public, continuous, peaceful and
adverse possession of the subject parcels of
The need, therefore, to determine once and land under bona fide claims of ownership for
for all whether the lands subject of more than 80 years. The CA affirmed the CFI’s
petitioner's reversion efforts are foreshore decision, holding that the classification of the
lands constitutes good and sufficient cause lotsas timberland by the Director of Forestry
for relaxing procedural rules and granting cannot prevail in the absence of proof that the
the third and fourth motions for extension said lots are indeed more valuable as forest
to file appellant's brief. Petitioner's appeal land than as agricultural land, citing as
presents an exceptional circumstance authority the case of Ankron vs. Government of
impressed with public interest and must the Philippine Islands (40 Phil.10).
then be given due course. Issue/s:
Whether or not the possession of forestlands
In the case at bar, the need to determine or timberlands for 80 years can ripen to private
once and for all whether the lands subject ownership.
of petitioners reversion efforts are Ruling:
foreshore lands constitutes good and No. The Court ruled that possession of
sufficient cause for relaxing the procedural forestlands, however long, cannot ripe ninto
rules and granting the third and fourth private ownership. It emphasized that a
motions for extensions to file appellants positive act of the government, particularly the
brief. Petitioner Republics appeal presented Executive Department is needed to declassify
an exceptional circumstance impressed land, which is classified as forest, and to
with public interest which in the Courts convert it into alienable or disposable land for
discretion must be given due course. agricultural or other purposes before
registration of which may proceed. The Court,
citing various cases, stated that a parcel of
forestland is within the exclusive jurisdiction of
19 Director of Lands vs Court of Appeals the Bureau of Forestry, an office under the
and Bisnar (GR No. 83609, 10/26/1989) Executive Department, and beyond the power
and jurisdiction of the cadastral court to
CLARK, Immaculate Gonzales register under the Torrens System.

Director of Lands v. Court of Appeals (178


SCRA 708) 20 Montano vs Insular Government (GR
No. 3714, 1//26/1909) CEBALLOS, Jesus
Facts: Cadavez
On July 20,1976, Ibarra and Amelia Bisnar filed
their joint application for the registration of two Facts:
parcels of land, located in the province of
Capiz, in the CFI of Capiz. They claimed that Montano applied for land registration of
they inherited those parcels of land. The a parcel of land used for fishery in Libis,
Director of Lands and Director of the Bureau of Caloocan. This was opposed to by the
Forest Development opposed the application Solicitor-General (Sol. Gen.) contending that
on the ground that said parcels of land were the land is owned by the US Government as
part of a timberland, a public dominion, so it well as Obras Pias de la Sagrda Mitra as it was
cannot be the subject of the registration the absolute owner of the lot.
proceedings. After the hearing, the CFI ordered
the registration of the title of the lots in the The Court of Land Registration granted
names of the applicants, herein private the application of Montano. Only the Sol. Gen.
respondents after finding that the applicants appealed the decision.

MARIA ANNY G. YANONG JMC COLLEGE OF LAW- LLB 5YR PROGRAM Page 22
NATRES CASE DIGEST

§ On the basis of the order of award of the


Issue: Director of Lands the Undersecretary of
Agriculture and Natural Resources issued
Whether the lands used as a fishery , for on August 26, 1959, Miscellaneous Sales
the growth of nipa, and as salt deposits, inland Patent No. V-1209 pursuant to which OCT
some distance from the sea, and asserted, No. 510 was issued by the Register of
though not clearly proved to be overflowed at Deeds of Naga City on October 21, 1959.
high tide could be registered as private
property. § Land in question is not a private property
as the Director of Lands and the Secretary
Held: of Agriculture and Natural Resources have
always sustained the public character for
Yes, it can. having been formed by reclamation (as
opposed to peittioners contention that it is
The Supreme Court held that since fish accretion)
ponds are not classified as forest or mineral
lands, it must necessarily be agricultural land. § The only remedy: action for
The Court noted that before Montano owned reconveyance on the ground of fraud - But
the lot, bacawan had been sown and there was no fraud in this case
propagated in the mud by the owner.
ISSUES:
The Court also noted that government 1. W/N Lee Hong Kok can question the
land is NOT similar to public domain or public grant. - NO
land. Government land includes not only public
domain, but also other lands of the 2. W/N David has original acquisition of
Government already reserved or developed to title. - YES
public use or subject to private right.
Government owns real estate which is
part of public lands and other real estate which
is not part thereof. There is much real property HELD: Court of Appeals Affirmed. (no legal
belonging to the Government which is not justification for nullifying the right of David to
affected by statutes for the settlement, the disputed lot arising from the grant made in
prescription or sale of public lands. Examples his favor by respondent officials)
in point are properties occupied by public § Only the Government, represented by
buildings or devoted to municipal or other the Director of Lands, or the Secretary of
governmental uses. Agriculture and Natural Resources, can
bring an action to cancel a void certificate
of title issued pursuant to a void patent. The
21 Lee Hong Kokvs David (GR No.G.R. legality of the grant is a question between
No. L-30389, 12/27/1972) CAMASURA, the grantee and the government. Private
Jayson Ug-ay parties like the plaintiffs cannot claim that
the patent and title issued for the land
FACTS: involved are void since they are not the
§ Aniano David acquired lawful title registered owners thereof nor had they
pursuant to his miscellaneous sales been declared as owners in the cadastral
application in accordance with which an proceedings of Naga Cadastre after
order of award and for issuance of a sales claiming it as their private property.
patent (*similar to public auction) was made
by the Director of Lands on June 18, 1958, § Well-settled Rule : no public land can be
covering Lot 2892. acquired by private persons without any

MARIA ANNY G. YANONG JMC COLLEGE OF LAW- LLB 5YR PROGRAM Page 23
NATRES CASE DIGEST

grant, express or implied, from the The application of TCT was brought by the
government operation of the Land Registration of Authority
as amended by the Property Registration
§ Cabacug v. Lao: holder of a land Decree No. 1529 proceeding to declare:
acquired under a free patent is more
favorably situated than that of an owner of 1. That its applicants –the Menguito’s, are
registered property. Not only does a free owners, in fee simple, these 11 parcels of land.
patent have a force and effect of a Torrens 2. Listing the applicants lot title numbers,
Title, but in addition the person to whom it attendant documents given with their
is granted has likewise in his favor the right respective Technical Descriptions.
to repurchase within a period of 5 years.
For the said application, the RTC of Pasig
§ Imperium v. Dominium issued a notice of its initial hearing against the
whole world publishing the same at Abante
1. Imperium - government authority Tabloid on April 5, 1989.
possessed by the state which is
appropriately embraced in the concept of Seven days earlier, however, March 30,
sovereignty 1989, the Office of Solicitor General, filed its
contention as:
2. Dominium - capacity to own or 1. Applicants nor its predecessor’s
acquire property. The use of this term is in interest were neither in open,
appropriate with reference to lands held continuous, exclusive, adverse and
by the state in its proprietary character. In notorious possession or
such capacity, it may provide for the occupation of the land they applied
exploitation and use of lands and other for since 1945.
natural resources, including their 2. Applicants don’t have
disposition, except as limited by the competent and sufficient evidence
Constitution. of bonafide acquisition –without
open, uninterrupted-continuous,
22 Director of Lands vs CA and Valeriano exclusive, adverse or notorious
(GR No. 58867, 6/22/1984) occupation of the lot in the concept
CABALLERO, Jeremiah Napalan of the owner and so appearing not
genuine or indicative of pretended
23 Menguito vs Republic (GR No. 134308, possession.
12/14/2000) BENITEZ, Winnie L 3. Titling from fee simple with
Spanish grant title were not
Menguito v. Republic anymore available after Feb 16,
[G.R. No. 134308. December 14, 2000] 1976 as required by PD 892.
4. That the said parcel of
FACTS: land applied for is part of the public
domain and belonging to the
A petition for review assailing the Court of Republic of the Philippines –and
Appeals Sept 30, 1997 decision against the not subject to private appropriation.
Menguito’s –the petitioners, with promulgated
resolution 10-Mos. later reversing the decision The OSG thus stated its valid opposition on the
of the Regional Trial Court of Pasig City. The presented documents by the applicants leaving
RTC decision confirmed the application for the its prayers that said application be denied and
titling of the parcel of land with aggregate area land in question be reverted to the ownership
of 2112 sqm located at Brgy Ususan, Taguig, of the Republic of the Philippines.
Metro Manila, in favor of the Menguito’s.

MARIA ANNY G. YANONG JMC COLLEGE OF LAW- LLB 5YR PROGRAM Page 24
NATRES CASE DIGEST

At the appellate court, the RTC’s decision -thus denied the petition and declared cost
favoring the registrations of the land applied against petitioner.
were reversed and thus this petition.

ISSUE: WON the CA’s did not err in its Note:


decision to reverse the trial court findings.
Menguito v. Republic:
HELD: No. The petition is without merit.
“Unless public land is shown to have been
The Court cited Sec. 48 of reclassified or alienated to a private person by
Commonwealth Act No. 141 as amended, the State, it remains part of the inalienable
provide the registration the registration of the public domain. Indeed, occupation thereof in
title of lands in this wise; the concept of owner, no matter how long,
cannot ripen into ownership and be registered
“(b) Who by themselves, or their predecessors as a title.
in interest have been in open, continuous,
exclusive, and notorious occupation of the land
in the concept of the owner.” 24 MesinavsSonza (GR No. L-14722,
5/25/1960) BANTUGAN
The Court thus observed that the petitioners
have insufficient evidence on this.” 25 Susi vsRazon (G.R. No. L-
24066December 9, 1925) BALUCANAG,
The Court likewise pointed that; April Gem Balansag

• Applicants to avail the OCT they must


overcome the presumption that the land FACTS:
they applied for forms part of the public
domain and if so, it has to be shown as re- On September 5, 1899, Valentin Susi bought a
classified or alienated to a private person parcel of land from Apolonio Garcia and Basilio
by the state. Mendoza. Prior to such purchase, the said
• Otherwise, the land remain inalienable sellers have been in an open, continuous,
public domain. adverse and public possession of the land for
• To overcome this, an incontrovertible or 19 years.
clear and convincing evidence must be After the purchase, Valentin Susi took
presented. possession of the land in an open, continuous,
• The court observed that applicants adverse and public manner and without
possession were tacked only as far back interruption. However on September 13, 1913,
as 1948 to its predecessors of interest –the the possession was interrupted when Angela
same must have shown that their Razon filed an action to recover possession of
predecessors in interest were in the land. The Court of First Instance dismissed
possession of the property by some 30- Angela Razon’s complaint.
years back or 1938 before WW-2. Despite the dismissal of the case, Angela
Razon filed for the purchase of the disputed
The court find the petitioners land with the Director of Lands. Valentin Susi
failed to show it even as they claimed that filed his opposition and asserted his right over
Cirilo Menguito once declared the land for tax the land. However, the Director of Land
purposes in 1943 –yet failed to show overruled the opposition and sold the land the
documentary evidence to support the claim. Angela Razon on the ground that the land was
still and public land and Valentin Susi does not
The court therefore find no have a title over it.
reason to modify the appellate court’s decision

MARIA ANNY G. YANONG JMC COLLEGE OF LAW- LLB 5YR PROGRAM Page 25
NATRES CASE DIGEST

ISSUE: Applicant Jose Lachica filed this application for


title to land on April 28, 1958 with the claim that
WON Valentin Susi already acquired title to the the land applied for was purchased by him and
land. his wife, Adela Raz from one Eulalio Raz. The
documents attached to the application are:
HELD: technical description, surveyor's certificate,
certification by the chief deputy assessor of
YES. Valentin Susi has been in possession of Aklan and the blue print of Psu-161277. The
the land in question openly, continuously, land applied for is residential.
adversely and publicly personally and through The initial hearing was held on and an order of
his predecessors for 45 years. This being the general default was issued but those who
case, the doctrine laid down by the Supreme presented their opposition, namely, Octabela
Court of United States in Cariño vs Alba Vda. De Raz, Manuel and Susana
Government of the Philippine Islands is Braulio, Jose Rago, representing Apolonia
applicable. Here, it held that To acquire a right Rebeco.
to a certificate of title over a land of the public Petitioners contended that they were the real
domain, under the provisions of Chapter VI of owners of the subject land, which they have
Act No. 926, as amended by Chapter VIII of been possession continuously, openly and
Act No. 2874, an open, adverse, public and peacefully under claim of ownership for not
continuous possession from July 26,1894, is less than 70 years together with their
sufficient, provided the possessor makes predecessor-in-interest, and the applicant is
application therefor under the provisions of only entitled to have the portion of 620 square
section 47 of Act No. 2874. The possessor meters which is contrary to his application of
under such circumstances acquires by 4,845 square meters.
operation of law, not only a right to a grant, but The trial court finds that Dr. Jose Lachica as
a grant of the government, and the actual the absolute owner in fee simple of the land
issuance of a title is not necessary in order that described in his application for its original
said grant may be sanctioned by the courts. registration in his name.
In the present case all the requirements for a
grant were complied with since Valentin Susi Issue:
has been in possession of the land in question WON the private respondent/applicant is
openly, continuously, adversely, and publicly, entitled to the confirmation of his ownership in
personally and through his predecessors, since fee simple for the 4, 845 square meter parcel
the year 1880, that is, for about forty-five years. of land he applied for.
By a legal fiction, Valentin Susi had acquired
the land in question by a grant of the State, it Held: NO
had already ceased to be the public domain In Section 48 of Commonwealth Act 141,
and had become private property, at least by (which is the law enforced during the filing of
presumption, of Valentin Susi, beyond the application) as amended by RA Nos. 1942 and
control of the Director of Lands. Consequently, 6236, which states that:
in selling the land in question to Angela Razon, Sec. 48. The following-described citizens of the
the Director of Lands disposed of a land over Philippines, occupying lands of the public
which he had no longer any title or control, and domain or claiming to own any such lands or
the sale thus made was void and of no effect, an interest therein, but whose titles have not
and Angela Razon did not thereby acquire any been perfected or completed, may apply to the
right. Court of First Instance of the province where
the land is located for confirmation of their
26 Alba vs Court of Appeals (GR No. claim and issuance of a certificate of title
120066, 9/9/1999) APURADA, Kathryne therefor, under the Land Registration Act, to
Vencio wit:
Facts:

MARIA ANNY G. YANONG JMC COLLEGE OF LAW- LLB 5YR PROGRAM Page 26
NATRES CASE DIGEST

(a) Those who prior to the transfer of a. as to the (a) of Section 48 of CA 141-
sovereignty from Spain to the United States respondent did present any imperfect title with
have applied for the purchase, composition or Spanish decree.
other form of grant of lands of the public b. as to the (b) of Section 48 of CA 141-
domain under the laws and royal decrees then prescription cannot be applied as there were
in force and have instituted and prosecuted the no express grant from the government that the
proceedings in connection therewith, but have subject land was amongst the agricultural land
with or without default upon their part, or for certified as alienable and disposable.
any other cause, not received title therefor, if Ultimately, No public land can be acquired by
such applicants or grantees and their heirs private persons without any grant, express or
have occupied and cultivated said lands implied from the government; it is
continuously since the filing of their indispensable that there be a showing of title
applications. 49 from the state.
(b) Those who by themselves or through their c. nor he is members as specified on the (c)
predecessors in interest have been in open, condition of CA 141.
continuous, exclusive and notorious
possession and occupation of agricultural
lands of the public domain under a bona fide 27 Republic vs Court of Appeals and
claim of ownership, for at least thirty years Naguit (GR No. 1/17/2005) ANTOPINA,
immediately preceding the filing of the Babielen Poliquit
application for confirmation of title except when Facts:
prevented by war or force majeure. These shall
be conclusively presumed to have performed On January 5, 1993, Naguit filed a petition for
all the conditions essential to a Government registration of title of a parcel of land. The
grant and shall be entitled to a certificate of title application sought a judicial confirmation of
under the provisions of this chapter. 50 imperfect title over the land.
(c) Members of the national cultural minorities
who by themselves or through their The public prosecutor, appearing for the
predecessors-in-interest have been in open, government, and Angeles opposed the petition.
continuous, exclusive and notorious The court issued an order of general default
possession and occupation of lands of the against the whole world except as to Angeles
public domain suitable to agriculture, whether and the government.
disposable or not, under a bona fide claim of
ownership for at least 30 years shall be entitled The evidence revealed that the subject parcel
to the rights granted in subsection (b) hereof. 51 of land was originally declared for taxation
A circumspect scrutiny of the assailed Decision purposes in the name of Urbano in 1945.
readily shows that in the affirming the ruling of Urbano executed a Deed of Quitclaim in favor
the trial court, the Court of Appeals relied on of the heirs of Maming, wherein he renounced
the provisions of Section 19 of Act 496 52 in all his rights to the subject property and
relation to the Civil Code's provision's on confirmed the sale made by his father to
prescription on the assumption that the subject Maming sometime in 1955 or 1956.
land is private land. Therein lies the flaw in the Subsequently, the heirs of Maming executed a
appellate court's postulate. The application for deed of absolute sale in favor of respondent
registration of private respondent is for judicial Naguit who thereupon started occupying the
confirmation of an imperfect title considering same.
that the land is presumed under the Regalian
Doctrine to be part of the public domain. Naguit constituted Blanco, Jr. as her attorney-
The private respondent failed to satisfy the in-fact and administrator. The administrator
Supreme Court that he complied the condition introduced improvements, planted trees in
set forth for the judicial confirmation of his title addition to existing coconut trees which were
for the following reasons, to wit; then 50 to 60 years old, and paid the

MARIA ANNY G. YANONG JMC COLLEGE OF LAW- LLB 5YR PROGRAM Page 27
NATRES CASE DIGEST

corresponding taxes due on the subject land. bona fide claim of ownership since June 12,
1945, or earlier.
Naguit and her predecessors-in-interest had
occupied the land openly and in the concept of (2) Those who have acquired ownership over
owner without any objection from any private private lands by prescription under the
person or even the government until she filed provisions of existing laws.
her application for registration.
There are three obvious requisites for the filing
The OSG argued that the property which is in of an application for registration of title under
open, continuous and exclusive possession Section 14(1) – that the property in question is
must first be alienable. Since the subject land alienable and disposable land of the public
was declared alienable only on October 15, domain; that the applicants by themselves or
1980, Naguit could not have maintained a bona through their predecessors-in-interest have
fide claim of ownership since June 12, 1945, been in open, continuous, exclusive and
as required by Section 14 of the Property notorious possession and occupation, and; that
Registration Decree, since prior to 1980, the such possession is under a bona fide claim of
land was not alienable or disposable. ownership since June 12, 1945 or earlier.

The OSG suggested an interpretation that all The OSG's interpretation would render
lands of the public domain which were not paragraph (1) of Section 14 virtually
declared alienable or disposable before June inoperative and even precludes the
12, 1945 would not be susceptible to original government from giving it effect even as it
registration, no matter the length of decides to reclassify public agricultural lands
unchallenged possession by the occupant. as alienable and disposable. The
unreasonableness of the situation would even
Issue: be aggravated considering that before June 12,
1945, the Philippines was not yet even
Whether or not it is necessary under Section considered an independent state.
14(1) of the Property Registration Decree that
the subject land be first classified as alienable The more reasonable interpretation of Section
and disposable before the applicant’s 14(1) is that it merely requires the property
possession under a bona fide claim of sought to be registered as already alienable
ownership could even start. and disposable at the time the application for
registration of title is filed. If the State, at the
Held: time the application is made, has not yet
deemed it proper to release the property for
Section 14 of the Property Registration Decree, alienation or disposition, the presumption is
governing original registration proceedings, that the government is still reserving the right
provides: to utilize the property; hence, the need to
preserve its ownership in the State irrespective
SECTION 14. Who may apply.— The following of the length of adverse possession even if in
persons may file in the proper Court of First good faith. However, if the property has
Instance an application for registration of title to already been classified as alienable and
land, whether personally or through their duly disposable, as it is in this case, then there is
authorized representatives: already an intention on the part of the State to
abdicate its exclusive prerogative over the
(1) those who by themselves or through their property.
predecessors-in-interest have been in open,
continuous, exclusive and notorious In this case, the 3 requisites for the filing of
possession and occupation of alienable and registration of title under Section 14(1) had
disposable lands of the public domain under a been met by Naguit. The parcel of land had

MARIA ANNY G. YANONG JMC COLLEGE OF LAW- LLB 5YR PROGRAM Page 28
NATRES CASE DIGEST

been declared alienable; Naguit and her Environment and Natural Resources (CENRO-
predecessors-in-interest had been in open, DENR), which stated that the subject
continuous, exclusive and notorious property was “verified to be within the Alienable
possession and occupation of the land or Disposable land per Land Classification
evidenced by the 50 to 60-year old trees at the Map No. 3013 established under Project No.
time she purchased the property; as well as the 20-A and approved as such under FAO 4-
tax declarations executed by the original owner 1656 on March 15, 1982.” On 3 December
Urbano in 1954, which strengthened one's 2002, the RTC approved the application for
bona fide claim of ownership. registration.

28 Heirs of Malabanan v. Republic, The Republic interposed an appeal to the


G.R. No. 179987, [April 29, 2009], 605 PHIL Court of Appeals, arguing that Malabanan had
244-326) JESURA failed to prove that the property belonged to
the alienable and disposable land of the public
domain, and that the RTC had erred in finding
FACTS: that he had been in possession of the
property in the manner and for the length of
On 20 February 1998, Mario Malabanan filed time required by law for confirmation of
an application for land registration before the imperfect title. On 23 February 2007, the Court
RTC of Cavite-Tagaytay, covering a parcel of of Appeals reversed the RTC ruling and
land situated in Silang Cavite, consisting of dismissed the application of Malabanan.
71,324 square meters. Malabanan claimed that ISSUES:
he had purchased the property from 1. In order that an alienable and disposable
Eduardo Velazco, and that he and his land of the public domain may be registered
predecessors-in-interest had been in open, under Section 14(1) of Presidential Decree No.
notorious, and continuous adverse and 1529, otherwise known as the Property
peaceful possession of the land for more than Registration Decree, should the land be
thirty classified as alienable and disposable as of
(30) years. Velazco testified that the property June
was originally belonged to a twenty-two 12, 1945 or is it sufficient that such
hectare property owned by his great- classification occur at any time prior to the filing
grandfather, Lino Velazco. Lino had four sons– of the
Benedicto, Gregorio, Eduardo and Esteban– applicant for registration provided that it is
the fourth being Aristedes’s grandfather. Upon established that the applicant has been in
Lino’s death, his four sons inherited the open, continuous, exclusive and notorious
property and divided it among themselves. But possession of the land under a bona fide claim
by of
1966, Esteban’s wife, Magdalena, had become ownership since June 12, 1945 or earlier?
the administrator of all the properties 2. For purposes of Section 14(2) of the
inherited by the Velazco sons from their father, Property Registration Decree may a parcel of
Lino. After the death of Esteban and land
Magdalena, their son Virgilio succeeded them classified as alienable and disposable be
in administering the properties, including Lot deemed private land and therefore susceptible
9864-A, which originally belonged to his uncle, to
Eduardo Velazco. It was this property that acquisition by prescription in accordance with
was sold by Eduardo Velazco to Malabanan. the Civil Code?
Among the evidence presented by Malabanan
during trial was a Certification dated 11 June
2001, issued by the Community Environment &
Natural Resources Office, Department of HELD:

MARIA ANNY G. YANONG JMC COLLEGE OF LAW- LLB 5YR PROGRAM Page 29
NATRES CASE DIGEST

The Petition is denied. ownership of patrimonial property by


prescription under the Civil Code is entitled to
(1) In connection with Section 14(1) of the secure
Property Registration Decree, Section 48(b) of registration thereof under Section 14(2) of the
the Public Land Act recognizes and confirms Property Registration Decree.
that “those who by themselves or through (b) There are two kinds of prescription by
their predecessors in interest have been in which patrimonial property may be acquired,
open, continuous, exclusive, and notorious oneordinary and other extraordinary. Under
possession and occupation of alienable and ordinary acquisitive prescription, a person
disposable lands of the public domain, under a acquires
bona fide claim of acquisition of ownership, ownership of a patrimonial property through
since June 12, 1945” have acquired ownership possession for at least ten (10) years, in good
of, and registrable title to, such lands based on faith and with just title. Under extraordinary
the length and quality of their possession. acquisitive prescription, a person’s
uninterrupted adverse possession of
(a) Since Section 48(b) merely requires patrimonial property for at least thirty (30)
possession since 12 June 1945 and does not years,
require that the lands should have been regardless of good faith or just title, ripens into
alienable and disposable during the entire ownership.
period of
possession, the possessor is entitled to secure It is clear that the evidence of petitioners is
judicial confirmation of his title thereto as insufficient to establish that Malabanan has
soon as it is declared alienable and disposable, acquired ownership over the subject property
subject to the timeframe imposed by under Section 48(b) of the Public Land Act.
Section 47 of the Public Land Act. There is no substantive evidence to establish
(b) The right to register granted under Section that Malabanan or petitioners as his
48(b) of the Public Land Act is further predecessors-in-interest have been in
confirmed by Section 14(1) of the Property possession of the property since 12 June 1945
Registration Decree. or
earlier. The earliest that petitioners can date
(2) In complying with Section 14(2) of the back their possession, according to their own
Property Registration Decree, consider that evidence—the Tax Declarations they
under the Civil Code, prescription is recognized presented in particular—is to the year 1948.
as a mode of acquiring ownership of Thus,
patrimonial property. However, public domain they cannot avail themselves of registration
lands become only patrimonial property not under Section 14(1) of the Property
only with a declaration that these are alienable Registration Decree.
or disposable. There must also be an
express government manifestation that the Neither can petitioners properly invoke Section
property is already patrimonial or no longer 14(2) as basis for registration. While the
retained for public service or the development subject property was declared as alienable or
of national wealth, under Article 422 of the disposable in 1982, there is no competent
Civil Code. And only when the property has evidence that is no longer intended for public
become patrimonial can the prescriptive period use service or for the development of the
for the acquisition of property of the public national evidence, conformably with Article 422
dominion begin to run. of the Civil Code. The classification of the
subject property as alienable and disposable
(a) Patrimonial property is private property of land of the public domain does not change its
the government. The person acquires status as property of the public dominion under
Article 420(2) of the Civil Code. Thus, it is
insusceptible to acquisition by prescription.

MARIA ANNY G. YANONG JMC COLLEGE OF LAW- LLB 5YR PROGRAM Page 30
NATRES CASE DIGEST

Grant, (3) Titulo de Compra or Title by


Purchase and (4) Informacion Posesoria or
Possessory Information title obtained under the
29 Heirs of Malabanan v. Republic, Spanish Mortgage Law or under the Royal
G.R. No. 179987, [September 3, 2013 Decree of January 26, 1889.
COSTAN
The decision of the CFI relied upon by
30 Palomo vs CA (GR No. 95608, petitioners were not signed by the judge but
1/21/1997) TAN, Cesnee Joyce V. were merely certified copies of notification to
Diego Palomo bearing the signature of the
Palomo v. CA (266 SCRA 392, G.R. No. Clerk of Court.
130906)
More importantly, the lands in question were
FACTS: not classified as alienable lands. Since the
In 1913, some 440,530 square meters of land lands were made part of a reservation for
in Albay were reserved for provincial park provincial park purposes, they form part of the
purposes by virtue of EO No. 40. Of said area, forest zone. Thus, they cannot be the valid
15 parcels of land were registered in the name subject of alienation.
of Diego Palomo by the Court of First Instance.

In 1937, Diego Palomo donated these lands to 31 Republic vs Bracewell[G.R. No.


his heirs Ignacio and Carmen Palomo. 107427. January 25, 2000] TACDER,
Claiming that the aforesaid original certificates Lovella Fe Madelo
of title were lost during the Japanese
occupation, Ignacio Palomo filed a petition for Facts:
reconstitution with the Court of First Instance of
Albay on May 30, 1950. The Register of The controversy involves a total of nine
Deeds of Albay issued Transfer Certificates of thousand six hundred fifty-seven (9,657)
Title Nos. 3911, 3912, 3913 and 3914 square meters of land located in Las Piñas,
sometime in October 1953. Metro Manila.

On July 10, 1954, President Magsaysay issued In 1908, Maria Cailles, married to James
Proclamation No. 47 converting the area Bracewell, Sr., who acquired the said parcels
covered by EO 40 into the Tiwi Hot Spring of land from the Dalandan and Jimenez
National Park. The Palomos contended that families of Las Piñas; after which
they have been in possession of the subject corresponding Tax Declarations were issued in
lands and have introduced improvements the name of Maria Cailles.
thereon.
On January 16, 1961, Maria Cailles sold the
ISSUE: Were the Original Certificate of Titles said parcels of land to her son, the petitioner,
issued to the petitioners valid? –NO by virtue of a Deed of Sale which was duly
annotated and registered with the Registry of
HELD Deeds of Pasig, Rizal. Tax Declarations were
Before the Treaty of Paris in 1899, the lands, thereafter issued in the name of petitioner,
whether agricultural, mineral, or forest were canceling the previous Tax Declarations issued
under the exclusive patrimony and dominion of to Maria Cailles.
the Spanish crown.Private ownership of land
could only be acquired through royal On September 19, 1963, petitioner filed before
concessions which were documented in the then Court of First Instance of Pasig, Rizal
various forms, such as (1) Titulo Real or Royal an action for confirmation of imperfect title
Grant," (2) Concession Especial or Special under Section 48 of Commonwealth Act No.

MARIA ANNY G. YANONG JMC COLLEGE OF LAW- LLB 5YR PROGRAM Page 31
NATRES CASE DIGEST

141. A similar situation in the case of Reyes v. Court


of Appeals, where a homestead patent issued
The Director of Lands, represented by the to the petitioners' predecessor-in-interest was
Solicitor General, opposed petitioner's cancelled on the ground that at the time it was
application on the grounds that neither he nor issued, the subject land was still part of the
his predecessors-in-interest possessed public domain.
sufficient title to the subject land nor have they
been in open, continuous, exclusive and In the said case, this Court ruled as follows —
notorious possession and occupation of the
same for at least thirty (30) years prior to the Under the Regalian doctrine, all lands of the
application, and that the subject land is part of public domain belong to the State, and that the
the public domain. State is the source of any asserted right to
ownership in land and charged with the
On May 3, 1989, the lower court issued an conservation of such patrimony. This same
Order granting the application of petitioner. The doctrine also states that all lands not otherwise
Solicitor General promptly appealed to appearing to be clearly within private
respondent Court which, on June 29, 1992, ownership are presumed to belong to the State
reversed and set aside the lower court's Order. (Director of Lands vs. Intermediate Appellate
It also denied petitioner's Motion for Court, 219 SCRA 340).
Reconsideration in its Resolution of September
30, 1992. Hence, the burden of proof in overcoming the
presumption of State ownership of lands of the
Issues: public domain is on the person applying for
registration. The applicant must show that the
a) Whether the failure of the petitioner to land subject of the application is alienable or
prosecute his action for an unreasonable disposable. These petitioners failed to do.
length of time?
The homestead patent was issued to
b) Whether the tax declarations attached to the petitioners' predecessor-in-interest, the subject
complaint do not constitute acquisition of the land belong to the inalienable and
lands applied for? undisposable portion of the public domain.
Thus, any title issued in their name by mistake
Held: or oversight is void ab initio because at the
time the homestead patent was issued to
The controversy is simple. On one hand, petitioners, as successors-in-interest of the
petitioner asserts his right of title to the subject original patent applicant, the Director of Lands
land under Section 48 (b) of Commonwealth was not then authorized to dispose of the same
Act No. 141, having by himself and through his because the area was not yet classified as
predecessors-in-interest been in open, disposable public land. Consequently, the title
continuous, exclusive and notorious issued to herein petitioners by the Bureau of
possession and occupation of the subject Lands is void ab initio.
parcels of land, under a bona fide claim of
acquisition or ownership, since 1908. On the Neither has petitioner shown proof that the
other hand, it is the respondents' position that subject Forestry Administrative Order
since the subject parcels of land were only recognizes private or vested rights under which
classified as alienable or disposable on March his case may fall. We only find on record the
27, 1972, petitioner did not have any title to Indorsement of the Bureau of Forest
confirm when he filed his application in 1963. Development from which no indication of such
Neither was the requisite thirty years exemption may be gleaned.
possession met.
Having found petitioner to have no cause of

MARIA ANNY G. YANONG JMC COLLEGE OF LAW- LLB 5YR PROGRAM Page 32
NATRES CASE DIGEST

action for his application for confirmation of for fictitious considerations in order to remove
imperfect title, we see no need to discuss the the same from the coverage of Sec. 38 of Act
other errors raised in this petition. 496, but in truth, buyers are mere dummies of
petitioners; hence, not purchasers for value.
The Court of First Instance denied this petition
32 Republic vs Court of Appeals and and on appeal, the CA affirmed the questioned
Bernabe (GR No. L-40402, 3/16/1987) decision. Petitioner’s Motion for
RAÑON, Rexie Monicimpo Reconsideration having been denied for lack of
merit; hence, this petition.
FACTS:
Lot No. 622 of the Mariveles Cadastre was ISSUE:
declared public land in a decision rendered WON the lots claimed by respondents could be
before the last war in Cadastral Case No. 19, legally be the subject of a juridical confirmation
LRC Cadastral Record No. 1097. On July 6, of Title under Section 48 (b) of Commonwealth
1965 such lot was segregated from the forest Act 141 as amended by Republic Act 1942.
zone and released and certified by the Bureau
of Forestry (BOF) as an agricultural Land for HELD:
disposition under the Public Land Act. On April No. The Supreme Court ruled that Sec. 48 (b)
26, 1967, Respondents filed in the CFI of of CA 141, as amended, applies exclusively to
Bataan a petition to reopen Cadastral Case public lands. Forest lands or areas covered
No. 19 to perfect their rights and register their with forests are excluded. Thus, possession of
titles to said lots. They alleged that they forest lands, however long cannot ripen into
acquired ownership and possession of said private ownership. A parcel of forest land is
parcels of land by purchase from the original within the exclusive jurisdiction of the Bureau
owners thereof, whose possession of the same of Forestry and beyond the power and
including that of the herein respondents, has jurisdiction of the cadastral court to register
always been continuous, open, active, under the Torrens System. Thus, even if the
exclusive, public, adverse and in the concept of reopening of the cadastral proceedings was at
owners for more than 30 years. The Director all possible, private respondents have not
of Forestry filed an opposition to the above qualified for a grant under Section 48 (b) of CA
petition but later withdrew the same upon 141. They can only be credited with 1 year, 9
verification of findings that this portion of the mos. and 20 days of possession and
timberland had already been released from the occupation of the lots involved, counted from
mass of the public forests. Subsequently, the July 6, 1965 when the lots involved had been
Acting Prov. Fiscal of Bataan, for and in behalf segregated from the forest zone and released
of the Director of Lands filed his opposition by the BOF as an agricultural land for
alleging that the land is still a Public Land and disposition under the Public Land Act. As
as such cannot be the subject of a land such, respondents and their predecessors in
registration proceeding under Act 496. The interest could not have possessed the lots for
lower court adjudicated in favor or respondent the required period of 30 years as disposable
Bernabes, finding that the latter have complied agricultural land.
with all the terms and conditions entitling them
to a grant. This decision having become final,
the Commissioner of Land Registration issued 33 Republic v. Bautista, Jr., G.R. No.
the corresponding decrees of registration. On 166890, [June 28, 2016]) PLAZA,
the other hand, petitioner DL through the Mariafe Manatad
Solicitor Gen. filed a petition for review of the
decrees. Afterwards, he filed an Amended FACTS:
Petition for Review, adding: that respondents Apolonio Bautista Jr. acquired lot 17078
executed simulated deeds of sale conveying through succession, when his father Apolonio
portions of the subject parcels to third parties Sr. died in 1987. He applied judicial

MARIA ANNY G. YANONG JMC COLLEGE OF LAW- LLB 5YR PROGRAM Page 33
NATRES CASE DIGEST

confirmation of imperfect title before MTC with


testimonial evidence that his father was in Facts:
possession of the said lot since 1969 and Manila International Airport Authority (MIAA)
acquired it from Jardin and Villanueva, through operates the Ninoy Aquino International Airport
notarized Deed of Absolute Sale dated (NAIA) Complex in Parañaque City. The Office
February 15, 1971 and May 25, 1973 of the Government Corporate Counsel (OGCC)
respectively. The MTC granted the application issued Opinion No. 061 which states that the
and declared him as the owner in fee simple of Local Government Code of 1991 withdrew the
the said land. exemption from real estate tax granted to
The Government of the Philippines appealed MIAA. Thus, MIAA negotiated with City of
before CA, contending that the testimony of Parañaque to pay the real estate tax imposed
Apolonio Jr. is an hearsay which should not be by the City. MIAA then paid some of the real
given probative value. The application of estate tax already due.
judicial confirmation of imperfect title must
comply with Sec. 48(b) of CA 141. The CA MIAA received Final Notices of Real Estate
affirmed the ruling of MTC, it pointed out that Tax Delinquency from the City of Parañaque
the Government of the Phil. did not timely for the taxable years 1992 to 2001 amounting
object to the evidence presented. to 624M. The City of Parañaque, through its
City Treasurer, also issued notices of levy and
ISSUE: W/N the grant of judicial confirmation warrants of levy on the Airport Lands and
of imperfect title to Apolonio Jr proper? Buildings. The Mayor of the City of Parañaque
threatened to sell at public auction the Airport
RULING: Lands and Buildings should MIAA fail to pay
The Supreme Court reversed the ruling of MTC the real estate tax delinquency. MIAA thus
and CA. It held that the requisite period of sought a clarification of OGCC Opinion No.
possession must conform with Sec 48(b) of 061. The OGCC issued Opinion No. 147
Public Land Act as amended by RA 1942 clarifying OGCC Opinion No. 061 and pointed
which provides that any person who applies for out that Local Government Code requires
judicial confirmation, he or his predecessor in persons exempt from real estate tax to show
interest must have been in open, continuous, proof of exemption. The OGCC opined that
exclusive and notorious possession and Section 21 of the MIAA Charter is the proof
occupation of alienable and disposable land of that MIAA is exempt from real estate tax. MIAA
public domain under the bonafide claim of then filed with the Court of Appeals an original
ownership at least 30 years since June 12, petition for prohibition and injunction, with
1945 or earlier. prayer for preliminary injunction or temporary
restraining order to restrain the City of
In the present case, the lower court relied only Parañaque from imposing real estate tax on,
on the testimony of Bautista Jr. that his father levying against, and auctioning for public sale
acquired the land and in possession since the Airport Lands and Buildings. However, this
1969. He failed to prove that the transferor had was dismissed due to because MIAA filed it
in open, continuous, exclusive and notorious beyond the 60-day reglementary period. The
possession of said land for at least 30 years Court of Appeals also denied MIAA's motion for
since June 12, 1945 or earlier. Bautista Jr. has reconsideration and supplemental motion for
no personal knowledge of these facts. Lack of reconsideration. Hence, MIAA filed a petition
this evidence does not give the court the right for review before the SC.
to grant a judicial confirmation of imperfect title
in favor of Bautista Jr. Issue:
WON the Land and Buildings of MIAA are part
34 MIAA vs Court of Appeals (GR of the public dominion and thus cannot be the
No. 155650, 7/20/2006) PELAEZ, Chester subject of levy and auction sale.
Bryan Pepito

MARIA ANNY G. YANONG JMC COLLEGE OF LAW- LLB 5YR PROGRAM Page 34
NATRES CASE DIGEST

Held: mentioned in said decision. On appeal,


Yes. The Court held that the land and buildings appellant, contented, among others that,
of MIAA are part of the public dominion. Since that portions of said land cannot be
the airport is devoted for public use, for the registered in accordance with the existing
domestic and international travel and Land Registration Law for the reason that
transportation. Even if MIAA charge fees, this they are manglares.
is for support of its operation and for regulation
and does not change the character of the land Issue: WON the land in question cannot
and buildings of MIAA as part of the public registered?
dominion.
Ruling:
As part of the public dominion the land and The mere fact that a tract of land has trees
buildings of MIAA are outside the commerce of upon it or has mineral within it is not of
man. To subject them to levy and public itself sufficient to declare that one is
auction is contrary to public policy. Unless the forestry land and the other, mineral land.
President issues a proclamation withdrawing There must be some proof of the extent and
the airport land and buildings from public use, present or future value of the forestry and
these properties remain to be of public of the minerals. While, as we have just said,
dominion and are inalienable. As long as the many definitions have been given for
land and buildings are for public use the "agriculture," "forestry," and "mineral"
ownership is with the Republic of the lands, and that in each case it is a question
Philippines. of fact, we think it is safe to say that in
order to be forestry or mineral land the
35 Roman Catholic Bishop of proof must show that it is more valuable for
Kalibovs Municipality of Buruanga (GR No. the forestry or the mineral which it contains
149145, 3/31/2006) PASAOL, Ric than it is for agricultural purposes. (Sec. 7,
Jason Patlingrao Act No. 1148.) It is not sufficient to show
that there exists some trees upon the land
36 Amunategui vs. Director of or that it bears some mineral. Land may be
Forestry (GR No. L-27873, 11/29/1983) classified as forestry or mineral today, and,
PAMISARAN, Excel Joy Gemota by reason of the exhaustion of the timber or
mineral, be classified as agricultural land
37 Ankronvs Government of the tomorrow. And vice-versa, by reason of the
Philippines (GR No. 14213, 8/23/1919) rapid growth of timber or the discovery of
NALLA, Glene Alacayde valuable minerals, lands classified as
agricultural today may be differently
Facts: This case involves an application for classified tomorrow. Each case must be
registration under the Torrens system of a decided upon the proof in that particular
certain parcel of land. The only opposition case, having regard for its present or future
of the said application which was value for one or the other purposes. We
presented by the Director of Landswas that believe, however, considering the fact that
the land in question was the property of the it is a matter of public knowledge that a
Government of the United States under the majority of the lands in the Philippine
control and administration of the Islands are agricultural lands, that the
Government of the Philippine Islands. The courts have a right to presume, in the
lower court ordered and decreed that said absence of evidence to the contrary, that in
parcel of land be registered in the name of each case the lands are agricultural lands
the said applicant, J. H. Ankron, subject, until the contrary is shown. Whatever the
however, to the right of the Government of land involved in a particular land
the Philippine Islands to open a road registration case is forestry or mineral land
thereon in the manner and conditions must, therefore, be a matter of proof. Its

MARIA ANNY G. YANONG JMC COLLEGE OF LAW- LLB 5YR PROGRAM Page 35
NATRES CASE DIGEST

superior value for one purpose or the other Barangay Castile, to which, by counterclaim,
is a question of fact to be settled by the Sta. Rosa sought ejectment against
proof in each particular case. The fact that respondents.
the land is a manglar [mangrove swamp] is
not sufficient for the courts to decide Respondents went to the DAR and filed a case
whether it is agricultural, forestry, or for compulsory acquisition of the Sta. Rosa
mineral land. It may perchance belong to Property under the Comprehensive Agrarian
one or the other of said classes of land. The Reform Program.
Government, in the first instance, under the
provisions of Act No. 1148, may, by Compulsory acquisition is the power of the
reservation, decide for itself what portions government to acquire private rights in land
of public land shall be considered forestry without the willing consent of its owner or
land, unless private interests have occupant in order to benefit the society.
intervened before such reservation is made.
In the latter case, whether the land is The said land was inspected by the Municipal
agricultural, forestry, or mineral, is a and Agrarian Reform Officer, and upon
question of proof. Until private interests consensus of the authorities concerned, they
have intervened, the Government, by virtue decided that the said land must be placed
of the terms of said Act (No. 1148), may under compulsory acquisition.
decide for itself what portions of the "public
domain" shall be set aside and reserved as Petitioners filed an objection on the ground
forestry or mineral land. that:

· The area is not appropriate for


38 Sta. Rosa Development agricultural purposes.
Corporation vs CA (GR No. 112526, · The area was rugged in terrain with
10/12/2001) MISTERIO, John Kessler slopes 18% and above. (which falls under
Sumauang the exception in compulsory acquisition of
CARP)
FACTS: · The occupants of the land were illegal
settlers or (squatters) who by no means are
The case is a petition regarding Department of entitled to the land as beneficiaries.
Agrarian Reform Adjudication Board’s
(DARAB) order of compulsory acquisition of ISSUE:
petitioner’s property under the Comprehensive · Whether or not the property in question
Agrarian Reform Program (CARP). is covered by CARP despite the fact that
the entire property formed part of a
Petitioner Sta. Rosa Development Corporation watershed area prior to the enactment of
(SRRDC), was the registered owner of two R.A No. 6657
parcel of land situated at Brgy. Casile, · Whether the petition of land
Cabuyao, Laguna. According to them, these conversion of the parcels of land may be
lands are watersheds which provide clean and granted?
potable (drinkable) water to the Canlubang
community and that 90 light industries are HELD:
located in that area.
Watershed is one of those enumerated by
They were alleging respondents usurped its CARP to be exempt from its coverage.
rights over their property thereby destroying
the ecosystem. Since the said land provides · Art. 67 of PD 1067 provides that Any
water to the residents, respondents sought an watershed or any area of land adjacent to
easement of a right of a way to and from any surface water or overlying any ground

MARIA ANNY G. YANONG JMC COLLEGE OF LAW- LLB 5YR PROGRAM Page 36
NATRES CASE DIGEST

water may be declared by the Department is inside IN-12 Mariquina Watershed.” The
of Natural resources as a protected area. Solicitor General filed oppositions to the
· Watersheds may be defined as an area application. Petitioners (Edna Collado and her
drained by a river and its tributaries and co-applicants) allege that they have occupied
enclosed by a boundary or divide which the Lot since time immemorial. Their
separates it from adjacent watersheds. possession has been open, public, notorious
and in the concept of owners. They paid all real
We cannot ignore the fact that the disputed estate taxes and submitted evidence to prove
parcels of land form a vital part of an area that that there have been 9 transfers of rights
need to be protected for watershed purposes. among them and their predecessors-in-
The protection of watersheds ensures an interest. RTC ruled in favor of the petitioners
adequate supply of water for future generations for having presented sufficient evidence to
and the control of flashfloods that not only establish registrable title over the property.
damage property but cause loss of lives.
Protection of watersheds is an ISSUE:
intergenerational responsibility that needs to be
answered now. (1) WON petitioners have registrable title over
the Lot. NO.
Although evidence of petitioners is strong, the
Supreme Court opines that the area must be (2) Did petitioners acquire private rights over
maintained for watershed purposes for the parcel of land prior to the issuance of EO
ecological and environmental considerations 33? NO.
despite the 88 families who are beneficiaries of
the CARP. It is important that a larger view of HELD:
the situation be taken because of the
thousands of residents downstream if the (1) Petitioners concede that the Lot is inside
watershed will not be protected and maintained the literal description of Marikina Watershed
for its natural purpose. Reservation (MWR). Their main claim over the
Lot is that “all Presidential proclamations like
Despite Supreme Court’s strong opinion of the proclamation setting aside the MWR are
protection of watersheds as an subject to private rights.” EO 33 (which
intergenerational responsibility, they, however established the MWR) has a saving clause that
ordered to DARAB to conduct a re-evaluation the reservations are “subject to existing private
of the case since the said land falls under rights, if any there be.”
exception.
Under the Regalian Doctrine, all lands not
otherwise appearing to be clearly within private
ownership are presumed to belong to the
State. The Spaniards first introduced the
doctrine to the Philippines through the Laws of
39 Collado vs Court of Appeals (GR the Indies and the Royal Cedulas, specifically,
No. 107764, 10/4/2002) LOZADA, Leah Law 14, Title 12, Book 4 of the Novisima
Amaya Recopilacion de Leyes de las Indias which laid
the foundation that "all lands that were not
FACTS: acquired from the Government, either by
purchase or by grant, belong to the public
Petitioner Edna T. Collado filed with the domain." Upon the Spanish conquest of the
land registration court an application for Philippines, ownership of all "lands, territories
registration of a parcel of land (“Lot”), situated and possessions" in the Philippines passed to
in Antipolo Rizal. Attached to the application the Spanish Crown.
was a technical description, stating “this survey

MARIA ANNY G. YANONG JMC COLLEGE OF LAW- LLB 5YR PROGRAM Page 37
NATRES CASE DIGEST

The Laws of the Indies were followed by the patrimonial property of the government and the
Ley Hipotecaria or the Mortgage Law of 1893. friar lands."
The Spanish Mortgage Law provided for the
systematic registration of titles and deeds as Thus, it is plain error for petitioners to argue
well as possessory claims. The Royal Decree that under the Philippine Bill of 1902 and Public
of 1894 or the "Maura Law" partly amended the Land Act No. 926, mere possession by private
Mortgage Law as well as the Law of the Indies. individuals of lands creates the legal
The Maura Law was the last Spanish land law presumption that the lands are alienable and
promulgated in the Philippines. It required the disposable.
"adjustment" or registration of all agricultural
lands, otherwise the lands would revert to the Both the 1935 and 1973 Constitutions
state. prohibited the alienation of all natural
resources except agricultural lands of the
Four years later, Spain ceded to the public domain. The 1987 Constitution
government of the United States all rights, readopted this policy. Indeed, all lands of the
interests and claims over the national territory public domain as well as all natural resources
of the Philippine Islands through the Treaty of enumerated in the Philippine Constitution
Paris of December 10, 1898. In 1903, the belong to the State.
United States colonial government, through the
Philippine Commission, passed Act No. 926, Watershed Reservation is a Natural Resource:
the first Public Land Act, which was described The term "natural resource" includes "not only
as follows: timber, gas, oil coal, minerals, lakes, and
submerged lands, but also, features which
"Act No. 926, the first Public Land Act, was supply a human need and contribute to the
passed in pursuance of the provisions of the health, welfare, and benefit of a community,
Philippine Bill of 1902. The law governed the and are essential to the well-being thereof and
disposition of lands of the public domain. It proper enjoyment of property devoted to park
prescribed rules and regulations for the and recreational purposes."
homesteading, selling and leasing of portions
of the public domain of the Philippine Islands, (2) An applicant must overcome the
and prescribed the terms and conditions to presumption that the land he is applying for is
enable persons to perfect their titles to public part of the public domain and that he has an
lands in the Islands. It also provided for the interest to warrant registration in his name
"issuance of patents to certain native settlers arising from an imperfect title (may have been
upon public lands," for the establishment of derived from old Spanish grants or titles). In
town sites and sale of lots therein, for the the case at bar, petitioners were unable to
completion of imperfect titles, and for the acquire a valid and enforceable right or title
cancellation or confirmation of Spanish because of the failure to complete the required
concessions and grants in the Islands." In period of possession (at least 30 years).
short, the Public Land Act operated on the
assumption that title to public lands in the Assuming that the Lot was alienable and
Name: I. Concept of Jura Regalia Natural disposable land prior to the issuance of EO 33
Resources First Set_ Philippine Islands in 1904, EO 33 reserved the Lot as a
remained in the government; and that the watershed. Since then, the Lot became non-
government’s title to public land sprung from disposable and inalienable public land. At the
the Treaty of Paris and other subsequent time petitioners filed their application on April
treaties between Spain and the United States. 25, 1985, the Lot has been reserved as a
The term "public land" referred to all lands of watershed under EO 33 for 81 years prior to
the public domain whose title still remained in the filing of petitioners’ application.
the government and are thrown open to private
appropriation and settlement, and excluded the

MARIA ANNY G. YANONG JMC COLLEGE OF LAW- LLB 5YR PROGRAM Page 38
NATRES CASE DIGEST

40 Director of Forestry vs. Villareal DIRECTOR OF FORESTRY v. VILLAREAL


(GR No. L-32266, 2/27/1989) LARANAS, G.R. No. L-32266 | February 27, 1989 | Cruz,
Rascille Mae Dagoon J.

FACTS: The said land consists of 178,113 assuming the requisite conditions, to justify our
square meters of mangrove swamps located in judicial intervention and scrutiny. The law is
the municipality of Sapian, Capiz. Ruperto thus presumed valid and so must be
Villareal applied for its registration on January respected. We repeat our statement in
25, 1949, alleging that he and his the Amunategui case that the classification of
predecessors-in-interest had been in mangrove swamps as forest lands is
possession of the land for more than forty descriptive of its legal nature or status and
years. He was opposed by several persons, does not have to be descriptive of what the
including the petitioner on behalf of the land actually looks like. That determination
Republic of the Philippines. After trial, the having been made and no cogent argument
application was approved by the Court of First having been raised to annul it, we have no duty
Instance of Capiz. The decision was affirmed as judges but to apply it.
by the Court of Appeals. The Director of
Forestry then came to this Court in a petition It follows from all this that the land under
for review on certiorari claiming that the land in contention being admittedly a part of the
dispute was forestal in nature and not subject mangrove swamps of Sapian, and for which a
to private appropriation. He asks that the minor forest license had in fact been issued by
registration be reversed. It is undisputed by the the Bureau of Forestry from 1920 to 1950, it
parties that the land in dispute is a mangrove must be considered forest land. It could
land HOWEVER the legal nature of mangrove therefore not be the subject of the adverse
swamps or manglares are still in contention. possession and consequent ownership claimed
Director of Forestry claims that it is forestall by the private respondent in support of his
and is not disposable. On the other hand, application for registration. To be so, it had first
Private respondents insists that it is alienable to be released as forest land and reclassified
as agricultural land. as agricultural land pursuant to the certification
the Director of Forestry may issue under
ISSUES: Are mangrove swamps classified as Section 1827 of the Revised Administrative
public forest lands? Code.

RULING: YES. Part of our public forest lands, The Respondent even showed, a survey of the
they are not alienable under the Constitution or land and its tax declaration to support its claim,
are they considered public agricultural lands; however the court held that the same is
they may be acquired under private ownership. insufficient especially now that the land is a
forest land.
Mangrove swamps or manglares should be
understood as comprised within the public WHEREFORE, the decision of the Court of
forests of the Philippines as defined in the Appeals is SET ASIDE and the application for
aforecited Section 1820 of the Administrative registration of title of private respondent is
Code of 1917. The legislature having so DISMISSED, with cost against him. This
determined, we have no authority to ignore or decision is immediately executory.
modify its decision, and in effect veto it, in the
exercise of our own discretion. The statutory 41 Atok-Big Wedge Mining
definition remains unchanged to date and, no Corporation vs Court of Appeals, GR No.
less noteworthy, is accepted and invoked by 88883, 1/18/1991) INOK, Erick Jay
the executive department. More importantly, Noro
the said provision has not been challenged as G.R. No. 88883 January 18, 1991
arbitrary or unrealistic or unconstitutional

MARIA ANNY G. YANONG JMC COLLEGE OF LAW- LLB 5YR PROGRAM Page 39
NATRES CASE DIGEST

FACTS:
On January 1984, the security guards of Atok
Fredia Mineral claim of about nine (9) hectares informed Feliciano Reyes, Security Officer of
situated in Tuding, Itogon, Benguet, was Atok, that a construction was being undertaken
located sometime between December 25, 1930 at the area of the Fredia mineral claim by
and December 31, 1930, a period of six (6) private respondent Liwan Consi. Feliciano
days, by A.I. Reynolds in accordance with the Reyes instructed the cashier to go and take
provisions of the Act of Congress of July 1, pictures of the construction. Feliciano Reyes
1902, better known as the Philippine Bill of himself and other security guards went to the
1902, in a so-called Declaration of Location. place of the construction to verify and then to
the police to report the matter.
The said Declaration of Location of mineral
claim was duly recorded in the Office of the On March 1, 1984, Atok filed a complaint for
Mining Recorder sometime on January 2, forcible entry and detainer against Liwan Consi
1931. Fredia mineral claim, together with other , which was dismissed after due hearing by the
mineral claims, was sold by A.I. Reynolds to MTC of Itogon in favor of Liwan Consi.
Big Wedge Mining Company, the earlier Petitioner ATOK appealed to the RTC of
corporate name of Atok Big Wedge Mining Baguio, which reversed the decision of the
Company, Inc. (Atok for short; herein MTC, ordering defendant Liwan Consi to
petitioner) in a Deed of Sale executed on vacate the premises of the Fredia Mineral
November 2, 1931. Since then petitioner Atok claim, restoring possession thereof to the
has been in continuous and exclusive plaintiff Atok Big Wedge Mining Company.
ownership and possession of said claim up to Defendant Liwan Cosi was further ordered to
the present . remove and demolish the house he
constructed in the premises of the land of
Atok has paid the realty taxes and occupation Fredia Mineral claim.
fees for the Fredia mineral claim. The Fredia
mineral claim together with other mineral In a petition for review filed by Liwan Consi
claims owned by Atok has been declared with the CA, the CA rendered its decision
under Tax Declaration No. 9535 and that in dismissing the subject forcible entry action, and
view of Presidential Decree No. 1214 an further rule in part that: Liwan Consi had a
application for lease was filed by Atok covering possessory right over the property which may
the Fredia mineral claim. mature into ownership on the basis of long-
term possession under the Public Land Law.
On the other hand, private respondent Liwan Thus, it held that both Consi and ATOK are of
Consi has a lot below the land of a certain Mr. equal footing with regards to the subject lot,
Acay at Tuding Slide, Itogon, Benguet. He holding possessory titles to the land. The
constructed a house thereon sometime in petitioner through its long term occupancy
1964. The lot is covered by Tax Declaration while respondent mining firm being the claim
No. 9462. When he first constructed his house locator and applicant for lease on the mineral
below the lot of Mr. Acay he was told that it claim.
was not necessary for him to obtain a building
permit as it was only a nipa hut. And no one ATOK filed a motion for reconsideration, which
prohibited him from entering the land so he was denied by the CA. Hence, this petition.
was constructing a house thereon. It was only
in January 1984 when private respondent ISSUE:
Consi repaired the said house that people Whether or not an individual's long term
came to take pictures and told him that the lot occupation of land of the public domain vests
belongs to Atok. Private respondent Consi has him with such rights over the same as to defeat
been paying taxes on said land which his the rights of the owner of that claim.
father before him had occupied .

MARIA ANNY G. YANONG JMC COLLEGE OF LAW- LLB 5YR PROGRAM Page 40
NATRES CASE DIGEST

HELD:
On the matter of possession, private
It is of no importance whether Benguet and respondent contends that his predecessor-in-
Atok had secured a patent for as held in the interest has been in possession of said lot
Gold Creek Mining Corporation case, for all even before the war and has in fact cultivated
physical purposes of ownership, the owner is the same. Since the subject lot is mineral land,
not required to secure a patent as long as he private respondent's possession of the subject
complies with the provisions of the mining lot no matter how long did not confer upon him
laws; his possessory right, for all practical possessory rights over the same.
purposes of ownership, is as good as though
secured by patent (Republic v. Court of Furthermore, Article 538 of the New Civil Code
Appeals, 160 SCRA 228 [1988]). provides:

In the case at bar, the evidence on record Art. 538. Possession as a fact cannot be
pointed that the petitioner Atok has faithfully recognized at the same time in two different
complied with all the requirements of the law personalities except in the cases of co-
regarding the maintenance of the said Fredia possession. Should a question arise regarding
Mineral Claim. the fact of possession, the present possessor
shall be preferred; if there are two possessors,
The perfection of the mining claim converted the one longer in possession; if the dates of the
the property to mineral land and under the laws possession are the same, the one who
then in force removed it from the public presents a title; and if all these conditions are
domain. By such act, the locators acquired equal, the thing shall be placed in judicial
exclusive rights over the land, against even the deposit pending determination of its
government, without need of any further act possession or ownership through proper
such as the purchase of the land or obtaining proceedings.
of a patent over it. As the land had become the
private property of the locators, they had the Since 1931 up to the present, petitioner ATOK
right to transfer the same, as they did, to has been in continuous and exclusive
Benguet and Atok . possession of the Fredia mineral claim while
private respondent's possession started only
As in the instant petition, the record shows that sometime in 1964 when he constructed a
the lot in question was acquired through a house thereon. Clearly, ATOK has superior
Deed of Sale executed between Atok and possessory rights than private respondent,
Fredia Mineral Claim. Liwan Consi, the former being "the one longer
in possession."

It is, therefore, evident that Benguet and Atok It is therefore clear that from the legal
have exclusive rights to the property in viewpoint it was really petitioner who was in
question by virtue of their respective mining actual physical possession of the property.
claims which they validly acquired before the Having been deprived of this possession by the
Constitution of 1935 prohibited the alienation of private respondent, petitioner has every right to
all lands of the public domain except sue for ejectment.
agricultural lands, subject to vested rights
existing at the time of its adoption. The land With this ruling enunciated by the Court, it can
was not and could not have been transferred to further be declared and held that petitioner
the private respondents by virtue of acquisitive Atok has the exclusive right to the property in
prescription, nor could its use be shared question.
simultaneously by them and the mining
companies for agricultural and mineral
purposes (Ibid).

MARIA ANNY G. YANONG JMC COLLEGE OF LAW- LLB 5YR PROGRAM Page 41
NATRES CASE DIGEST

42 Palomovs Court of Appeals [G.R.


No. 95608. January 21, 1997] IMPIG, 45 SIAN Enterprise vs. FF Cruz (GR
Sychar M no. 146616, 8/31/2006) FERNANDEZ,
Pauline August Momongan
43 Republic vs Southside
Homeowners (GR No. 156951, 9/22/2006) SIAIN ENTERPRISES, INC. v. F.F. CRUZ &
HONTANOSAS, Luzviminda Nee CO., INC.
Corong
500 SCRA 406 (2006)
44 Republic vsAlagad (G.R. No. L-
66807 January 26, 1989) GURO, Farhana That the foreshore area had been reclaimed
FACTS: On or about October 11, 1951, does not remove it from its classification of
defendants filed an application for registration foreshore area subject to the preferential right
of their title over a parcel of land situated at to lease of the littoral owner.
Linga, Pila, Laguna, with an area of 8.1263
hectares, reflected in survey plan Psu-116971, Facts:
which was amended after the land was divided
into two parcels, namely, Lot 1 with an area of Western Visayas Industrial Corporation
5.2476 hectares and Lot 2 with an area of (WESVICO) filed a foreshore lease application
2.8421 hectares, reflected in survey plan Psu- over the foreshore land adjacent to certain lots
226971, amd. 2. registered in its name. It eventually withdrew
The Republic opposed the application on the the application and filed a petition for
stereo-typed ground that applicants and their registration over the same foreshore land with
predecessors have not been in possession of the then Court of First Instance of Iloilo. The
the land openly, continuously, publicly and case was, however, archived as WESVICO‘s
adversely under a bona fide claim of ownership representative could no longer be contacted,
since July 26, 1894 and the land has not and later on, WESVICO has ceased
ceased to be a part of the public domain. It operations.
appears that barrio folk also opposed the
application. F.F. Cruz & Co. (F.F. Cruz) filed with the
Bureau of Lands, Iloilo City a foreshore lease
Issue: WON that applicants and their application over a foreshore land, a portion of
predecessors have not been in possession of which is adjacent to the lot previously occupied
the land openly, continuously, publicly and by WESVICO. Sian Enterprises Inc. (SIAIN)
adversely under a bona fide claim of ownership purchased the properties previously owned by
since July 26, 1894 WESVICO from the Development Bank of the
Philippines. It subsequently filed a foreshore
HELD: lease application over the foreshore land
The case, then, has to be decided alongside adjacent to the properties it bought from DBP.
these principles and regretfully, the Court
cannot make a ruling, in the first place, Upon learning that 130 linear meters of the
because it is not a trier of facts, and in the foreshore land subject of F.F. Cruz’s foreshore
second, it is in possession of no evidence to lease application overlapped that covered by
assist it in arriving at a conclusive disposition 31 its foreshore lease application, SIAIN filed a
We therefore remand the case to the court a protest 8 alleging that it being the owner of the
quo to determine whether or not the property property adjoining the overlapping area, it
subject of controversy is foreshore. We, should be given preference in its lease.
consequently, reverse both the Court of
Appeals and the trial court and reinstate the F.F. Cruz, argued that SIAIN must not be given
Republic's complaint. preferential right since the area in dispute is
classified as ―reclaimed‖ and that the

MARIA ANNY G. YANONG JMC COLLEGE OF LAW- LLB 5YR PROGRAM Page 42
NATRES CASE DIGEST

ownership was not by means of accretion. This


argument has been sustained by the Land As correctly argued by SIAIN, were
Management Bureau. WESVICO‘s petition for registration which, as
stated earlier, was archived by the trial court,
Upon appeal to the DENR Secretary, SIAIN pursued but eventually denied, WESVICO
was upheld, declaring that there was no basis would not have been barred from filing anew a
to declare the area as ―reclaimed‖. F.F. Cruz foreshore lease application. Parenthetically,
however appealed to the Office of the the petition for registration of WESVICO was
President which overturned the decision of the archived not on account of lack of interest but
DENR Secretary and found that the area is because it ceased operations due to financial
reclaimed. On appeal, the Court of Appeals reasons.
affirmed the decision. Hence, the present
petition. SIAIN contends that the evidence
overwhelmingly proves that the disputed area 46 Director of Lands vs. Roman
is foreshore land and not reclaimed land which Catholic Archbishop of Manila (GR No. 14869)
thus entitles it preferential rights over the ENERO, Jomari Ivan Tagud

ISSUES: FACTS:
In 1913, cadastral proceedings were
Whether the disputed land is a ―foreshore‖ or conducted to settle the title to a considerable
―reclaimed‖ area tract of land in the Province of Rizal. The
Roman Catholic Archbishop of Manila (church)
HELD: and other private parties were claimants of 13
cadastral lots that comprised the contested
That the foreshore area had been reclaimed property. The lower court ruled in favor of the
does not remove it from its classification of private claimants. Upon appeal, the church
foreshore area subject to the preferential right invoked that the composition title of the church
to lease of the littoral owner. with the Spanish Government included the
subject property. The church then presented
It bears noting that it was not the reclamation one witness and rested. The private oppositors
that brought the disputed foreshore area into then called their respective witnesses to prove
existence. Such foreshore area existed even title by possession, and rested. The church
before F.F. Cruz undertook its reclamation. It thereafter made an offer to present additional
was ―formed by accretions or alluvial deposits testimony intended to show that the
due to the action of the sea.‖ Following possession of the private claimants had been
Santulan, the littoral owner has preferential interrupted and that it was merely possession
right to lease the same. through the tolerance of the church. However,
the counsel for the oppositors objected to the
Contrary to the ruling of the Office of the entry of additional testimonies which was
President, as affirmed by the appellate court, sustained.
littoral owner WESVICO cannot be considered
to have waived or abandoned its preferential ISSUE:
right to lease the disputed area when it Did the lower court err in refusing the entry the
subsequently filed an application for church’s additional testimonies?
registration thereover. For being a part of the
public domain, ownership of the area could not RULING:
be acquired by WESVICO. Its preferential right The Court ruled in the affirmative.
remained, however. Its move to have the
contested land titled in its name, albeit a faux The object of a cadastral petition is that the title
pas, in fact more than proves its interest to to the various lots embraced in the survey may
utilize it. be settled and adjudicated. It is in the nature of

MARIA ANNY G. YANONG JMC COLLEGE OF LAW- LLB 5YR PROGRAM Page 43
NATRES CASE DIGEST

a proceeding in rem, promoted by the Director together with proofs of notification of


of Lands, somewhat akin to a judicial inquiry acceptance to the donor, as ground for new
and investigation leading to a judicial decree. trial.
In one sense, there is no plaintiff and there is In July of 1918, or four months after the above-
no defendant. In another sense, the mentioned decision of this Court, petitioner
Government is the plaintiff and all the- herein brought another action for recovery of
claimants are defendants. the land against the same defendants in the
previous case.
As a general rule, courts should adhere to the The second suit was later dismissed by the
usual rules of practice, procedure, and Court of First Instance and transferred to
evidence that governs registration cadastral case No. 5 which included the
proceedings. However, in registration hacienda in question that had in the meantime
proceedings where so many parties are been subdivided into lots.
involved and action is taken quickly and The latter through counsel moved that
abruptly, opportunity should be given to parties Abellera's claim over the lots concerned be
to submit additional corroborative evidence in dismissed on the grounds of res judicata and
support of their claims of title, if the ends of prescription.
justice so require. This case was remanded
back to the lower court with the church being Issue:
allowed to admit additional testimonies in the Did the cadastral court, on the ground of
interest of justice and ascertainment of truth. res judicata, have any power to entertain the
JOM motion to dismiss Abellera's claim and bar him
from presenting evidence to prove his
ownership of these lots?
47 AbelleravsFarol (GR No. 48480,
7/30/1943) DALISAY, Armando, Jr. Ruling:
D Rule 132 of the Rules of Court provides:
These rules shall not apply to land registration,
FABIAN B. S. ABELLERA, petitioner, cadastral and election cases, naturalization
vs. and insolvency proceedings, and other cases
MEYNARDO M. FAROL, ET AL., respondents not herein provided for, except by analogy or in
a suppletory character and whenever
Facts: practicable and convenient.
Abellera, in a previous case concerning the The Rules of Court may be applied in cadastral
same real estate involved herein, sued cases when two conditions are present: (1)
Hermegildo Balanag and others who are either analogy or need to supplement the cadastral
the same parties in this case or the latter's law, and (2) practicability and convenience.
predecessors in interest, alleging ownership of The principal aim is to settle as much as
the land. But his complaint was dismissed by possible all disputes over land and to remove
the Court of First Instance on two grounds: all clouds over land titles, as far a practicable,
(1) prescription in favor of defendants; and in a community.
(2) the deed of donation of these lands to him To attain this purpose, the cadastral court
had not been formally accepted according to should allow all claimants ample freedom to
Article 633 of the Civil Code. ventilate whatever right they may assert over
Upon appeal to this Court, the judgment of the real estate, permitting them, in keeping with the
trial court was affirmed on the second ground law of evidence, to offer proofs in support of
aforementioned. It appears in that decision of their allegations.
this Court that after the perfection of the We are, therefore, of the opinion that while in a
appeal, Abellera executed a public document cadastral case res judicata is available to a
formally accepting the donation of the land, claimant in order to defeat the alleged rights of
and presented and deed of acceptance

MARIA ANNY G. YANONG JMC COLLEGE OF LAW- LLB 5YR PROGRAM Page 44
NATRES CASE DIGEST

another claimant, nevertheless prior judgment encumbrance in favor of the government or


can not set up in a motion to dismiss. any of its branches, units or institutions.
In a number of cases, we have consistently
48 Gayapanao vs. IAC (GR No. ruled that a sale of homestead within the five
68109, 7/17/1991) CLARK, Immaculate (5) year prohibitive period is void ab initio and
Gonzales the same cannot be ratified nor can it acquire
Gayapanao vs. IAC (GR No. 68109, validity through the passage of time.
7/17/1991)

FACTS: 49 Republic vs Garcia (GR No. L-


This is a petition for review on certiorari filed by 11597, 5/27/1959) CEBALLOS, Jesus
Severino Gayapanao and his siblings Cadavez
questioning the decision of the IAC in
upholding the validity of the sale of the land Facts:
between their father Constantino and their
sister. The 2 hectare land, subject of this case Garcia was granted a homestead patent
is part of 10 hectare homestead land registered for his 23.21 hectares of land. After 3 years
in the name of Constantino Gayapanao under and 3 months later (April 14, 1950), he sold 19
OCT. The final order of the Director of Lands hectares to the lot to several persons. For this
for the issuance of patent was issued on reason, the CFI of Cotabato decreed the
December 10, 1937. On November 15, 1938, reversion of the property in favor of the
Constantino Gayapanao executed a private Government for violating the selling the
deed entitled kasulatan ng bilihan in favor of property within the prohibitive period of 5 years
Simeona Gayapanao and his husband. from date of issuance under Sec. 118 of CA
141.
RTC’s decision: The contract of sale between
Simeona Gayapanao and her father is null and Garcia contends that he did not violate
void for having been executed with the 5 year the said provision since the sale was not
prohibitory period provided under Section 118 registered and that it was not the entire land
of the Public Land Law was sold.
CA’s decision: It reversed the decision of the
RTC and uphold the validity of the sale. Issue:

ISSUE: WON CA was correct in upholding the Whether or not Garcia violated the Sec.
validity of the sale? 118 of CA 141.

RULING: Held.
No, the provision of law which prohibits the
sale or encumbrance of the homestead within Yes, he did.
5 years after the grant is mandatory.
The Supreme Court held that it is
From the date of the approval of the application enough that the property, in whole or in part,
and for a term of five (5) years from and after was alienated or encumbered within the
the date of issuance of the patent or grant, prohibitive period except in favor of the
lands acquired under free patent or homestead Government, Sec. 118 of CA 141 partly
provisions cannot be subject to encumbrance provides: Except in favor of the Government or
or alienation, nor shall they become liable to any its branches, units, or institutions, lands
the satisfaction of any debt contracted prior to acquired under free patent or homestead
the expiration of said period. The only provision shall not be subject to encumbrance
exception mentioned by the law is the sale or of alienation from the date of the approval of
the application and for a term of five years from

MARIA ANNY G. YANONG JMC COLLEGE OF LAW- LLB 5YR PROGRAM Page 45
NATRES CASE DIGEST

and after the date of issuance of the patent or Realty insists that Lee Chuy verbally notified of
grant, nor shall they become liable to the the sale and was given a copy of the deed of
satisfaction of any debt contracted prior to the sale.
expiration of said period, but the improvements
or crops on the land may be mortgaged or On 13 November 1989 LEE CHUY REALTY
pledged to qualified persons, association, or filed a complaint for legal redemption against
corporations. MARC REALTY and consigned in court a
manager's check for 614,400. MARC REALTY
Such alienation is a sufficient cause for insisted that the complaint be dismissed for
reversion to the State of the whole grant. In failure to state a cause of action there being no
granting a homestead to an applicant, the law allegation of prior valid tender of payment or a
imposes as a condition that the land should not prior valid notice of consignation.
be encumbered, sold or alienated within five
years from the issuance of the patent. Sec. 124 On Dec 26, 1990, the trial court ruled in favour
of CA 141 provides: Any acquisition, of Lee Chuy Realty which stated that there was
conveyance, alienation, transfer, or other a valid tender of payment and consignation. It
contract made or executed in violation of any of also stated that neither a separate offer to
the provisions of section118, 120, 121, 122, redeem nor a formal notice of consignation is
and 123 of this Act shall be unlawful and null necessary for the reason that the filing of the
and void from its execution and shall produce action itself, within the period of redemption, is
the effect of annulling and cancelling the grant, equivalent to a formal offer to redeem.
title, patent, or permit originally issued,
recognized or confirmed, actually or On 1 February 1991 MARC REALTY filed a
presumptively, and cause the reversion of the Petition for Certiorari, Prohibition with
property and its improvements to the State. Temporary Restraining Order and/or Writ of
Preliminary Injunction which was referred to
50 Lee Chuy Realty Corp vs Court of the Court of Appeals. The CA reversed the
Appeals (GR No. 104114, 12/4/1995) decision of the lower court and ruled that "a
CAMASURA, Jayson Ug-ay prior tender or offer of redemption is a
prerequisite or precondition to the filing of an
Facts: action for legal redemption” and that "there
must be tender of the redemption price within
A valuable piece of land located at Meycauyan, the required period because the policy of the
Bulacan, with an area of 24,576 sq. m. and law is not to leave the purchaser's title in
covered by OCT No. 0-5290 is disputed by Lee uncertainty beyond the established 30-day
Chuy Realty Corporation and Marc Realty and period.
Development Corp. Such land was originally
co-owned by Ruben Jacinto(one-sixth), MARC REALTY contends that prior tender of
Dominador, Arsenio, Liwayway all surnamed payment is a condition precedent to the filing of
Bascara and Ernesto jacinto(collectively owned an action in court in order to validly exercise
the remaining five-sixths). the right of legal redemption. LEE CHUY
REALTY however argues that the filing of the
On Feb. 4, 1981, Ruben Jacinto sold his one- action itself is equivalent to a formal offer to
sixth pro-indiviso share to LEE CHUY REALTY redeem, which is a condition precedent to the
which was registered 30 April 1981. On 5 May valid exercise of the right of legal redemption.
1989 the Bascaras and Ernesto Jacinto also
sold their share to MARC REALTY which was Lee Chuy filed a motion for reconsideration but
registered on 16 October 1989. was denied
Lee Chuy Realty claims that it was never
informed of the existence of the sale between
Marc Realty and the Bascaras/Jacinto. Marc Issue:

MARIA ANNY G. YANONG JMC COLLEGE OF LAW- LLB 5YR PROGRAM Page 46
NATRES CASE DIGEST

owner or mortgagor, although perhaps with


WON a formal offer to redeem accompanied unequal force and effect since each is given a
with tender of payment a condition precedent fixed but different period. A co-owner desirous
to the filing of an action for the valid exercise of of exercising his right of legal redemption is
the right of legal redemption; is the filing of the given a period of thirty (30) days from notice of
action with consignation equivalent to a formal the sale within which to avail of the right to
offer to redeem redeem.15 Under the free patent or homestead
provisions of the Public Land Act a period of
Held: five (5) years from the date of conveyance is
No. provided,16 the five-year period to be reckoned
from the date of the sale and not from the date
In Hulganza v. Court of Appeals14 the Court, of registration in the office of the Register of
citing previous decisions, declared that the Deeds.17 The redemption of extrajudicially
formal offer to redeem, accompanied by a foreclosed properties, on the other hand, is
bona fide tender of the redemption price, within exercisable within one (1) year from the date of
the prescribed period is only essential to the auction sale as provided for in Act No.
preserve the right of redemption for future 3135.
enforcement beyond such period of redemption
and within the period prescribed for the action
by the statute of limitations. Where, as in the
instant case, the right to redeem is exercised 51 Ong Ching Po vs. Court of
through judicial action within the reglementary Appeals (239 SCRA 341)
period the formal offer to redeem, CABALLERO, Jeremiah Napalan
accompanied by a bona fide tender of the
redemption price, while proper, may be 52 Frenzel vs Catito (GR No.
unessential. The filing of the action itself is 143958, July 11, 2003) BENITEZ, Winnie L
equivalent to a formal offer to redeem.
ALFRED FRITZ FRENZEL, petitioner,
In sum, the formal offer to redeem is not a v.
distinct step or condition sine qua non to the EDERLINA P. CATITO, respondent.
filing of the action in Court for the valid [G.R. No. 143958. July 11, 2003]
exercise of the right of legal redemption. What
constitutes a condition precedent is either a CALLEJO, SR., J.:
formal offer to redeem or the filing of an action
in court together with the consignation of the FACTS:
redemption price within the reglementary
period. Petitioner – Alfred Fritz Frenzel, is an
Australian Electrical Engineer who once
The doctrine in Tolentino, Tioseco and worked with New Guinea Airlines, started doing
Belisario cases was jettisoned by the Court of business in the Philippines since 1974, and two
Appeals on the ground that they do not involve years later got married to a Filipina Teresita
legal redemption by a co-owner but by a Santos. They were separated bread and board
mortgagor. It concluded that the application of in 1981 and two years later met Ederlina Catito
the rules on legal redemption by a co-owner as a masseuse in one of night clubs in Sydney
differs from the legal redemption by a Australia
mortgagor. But the law does not distinguish;
neither should we. For sure, the principle in the Frenzel got enamored with Ederlina and
aforecited cases is applicable regardless of convinced her to setup wholesome business in
whether the redemptioner is a co-owner or a Manila for her own as he was willing to help
mortgagor. Public policy favors redemption her. Ederlina returned and later Alfred followed
regardless of whether the redemptioner is a co- and helped her setup a Edorial Beauty saloon

MARIA ANNY G. YANONG JMC COLLEGE OF LAW- LLB 5YR PROGRAM Page 47
NATRES CASE DIGEST

with his personal funds –let alone Ederlina as


sole registered owner. Alfred went back to No. The petition is Denied.
work in Papue New Guinea as airline pilot
while Ederlina too went to Germany to file her The Court ruled on evidence on record: to
divorce with her husband –Muller. wit;

When Alfred returned to the Philippines to live “The three parcels of land subject of the
with Ederlina–he bought house and lot or real complaint were not mortgaged to the petitioner
properties in QC for her with his own funds and by the owners thereof but were sold to the
agreeing to name it under Ederlina alone respondent as the vendee, albeit with the use
accepting that as a foreigner he is not allowed of the petitioners personal funds.” The sales of
to own a real estate property under the three parcels of land in favor of the petitioner
Philippine Laws. Alfred decided to stay for who is a foreigner is illegal per se. The
good in the Philippines and sold his personal transactions are void ab initio because they
properties in Australia with proceeds later used were entered into in violation of the
to buy other 2-more properties in Davao City – Constitution. Thus, to allow the petitioner to
again, in Ederlina’s name only. There was also recover the properties or the money used in
an occasion when Ederlina deposited some the purchase of the parcels of land would be
USD 250,000 in their joint-accounts. subversive of public policy.

Meanwhile, Ederlina’s divorce case with her This finds support under Section 7, Article XII
German husband was denied and with the of the 1987 Philippine Constitution which
prospect that she could be charged with states: “Save in cases of hereditary
bigamy along with Alfed who himself is still succession, no private lands shall be
married. This started their common law transferred or conveyed except to individuals,
marriage deterioration and soon enough Alfred corporations or associations qualified to
got penniless and filed his claim on the acquire or hold lands of the public domain.”
properties listed in Ederlina’s name alone as
well as charges the return of all other asset The court stated: MEMO CUM ALTERIUS
bought with his own funds. DETER DETREMENTO PROTEST (No person
should unjustly enrich himself at the expense
ISSUE: of another). An action for recovery of what has
been paid without just cause has been
Will the good faith doctrine and applicable designated as an accion in rem verso.
Articles in Civil code 1416, In combination
with RA Nos. 133, 4381 and Rep. Act No. This provision does not apply if, as in this case,
4882, the proceeds of the sale would be the action is proscribed by the Constitution or
remitted to him, by way of refund for the money by the application of the pari delicto doctrine.
he used to purchase the said properties. To
bar the petitioner from recovering the subject It may be unfair and unjust to bar the petitioner
properties, or at the very least, the money used from filing an accion in rem verso over the
for the purchase and to bar the petitioner to the subject properties, or from recovering the
proceeds thereof is to enrich the respondent at money he paid for the said properties, but, as
his expense, and can this withstand the Lord Mansfield stated in the early case of
Constitutional proscription on the property Holman vs. Johnson. ”The objection that a
ownership for the aliens in the Philippines? contract is immoral or illegal as between the
plaintiff and the defendant, sounds at all times
Hence, this petition, after successive defeat very ill in the mouth of the defendant. It is not
from lower and appellate court. for his sake, however, that the objection is ever
allowed; but it is founded in general principles
HELD. of policy, which the defendant has the

MARIA ANNY G. YANONG JMC COLLEGE OF LAW- LLB 5YR PROGRAM Page 48
NATRES CASE DIGEST

advantage of, contrary to the real justice, as right on the said property cannot be
between him and the plaintiff, therefore, the sustained. Thus, in the instant case,
petition is bereft of merits. respondent cannot seek reimbursement on
the ground of equity where it is clear that
he willingly and knowingly bought the
53 Muller vs. Muller (GR no. 149615, property despite the constitutional
August 29, 2006) BALUCANAG, prohibition. To allow reimbursement would
April Gem Balansag in effect permit respondent to enjoy the
fruits of a property which he is not allowed
FACTS: to own. Thus, it is likewise proscribed by
Elena Buenaventura Muller and Helmut law.
Muller are husband and wife in this case.
They wed and resided in Germany until they 54 Lee vs. Director of Lands (GR
decided to permanently reside in the No. 128195, October 3, 2001)
Philippines in 1992. By this time, they APURADA, Kathryne Vencio
bought a house in Antipolo, Rizal using the Facts:
proceeds that they got from selling the Sometime in March 1936, Carmen, Francisco,
house the Helmut Muller inherited from his Jr., Ramon, Lourdes, Mercedes, Concepcion,
parents in Germany. Mariano, Jose, Loreto, Manuel, Rizal, and
The marriage, however, did not last. They Jimmy, all surnamed Dinglasan sold to Lee
were eventually separated and Helmut Liong, A Chinese citizen, a parcel of land with
Muller filed for separation of their an approximate area of 1,631 square meters,
properties. The trial court then dissolved designed as lot 398 and covered by Original
the absolute community of property and Certificate of Title No. 3389.
ordered the equal partition of their personal However, in 1948, the former owners filed with
properties located within the country, the Court of First Instance, Capiz an action
excluding those acquired by gratuitous title against the heirs of Lee Liong for annulment of
during marriage. As to the Antipolo sale and recovery of land. The plaintiffs
property, the court held that it was acquired assailed the validity of the sale because of the
using the paraphernal funds of Helmut, constitutional prohibition against aliens
however, he cannot recover said property, acquiring ownership of private agriculture land,
nor have a right to recover the funds used including residential, commercial or industrial
to buy it since it was purchased in violation land.
of Section 7, Article XII of the Constitution. Plaintiffs appealed to the Supreme Court and
Upon appeal, the Court of Appeals granted ruled thus: pari delicto (in sales of real estate to
Helmut Muller’s prayer for reimbursement aliens incapable of holding title thereto by
for the Antipolo property. virtue of provision of the Constitution, both the
vendor and vendee are deemed to have
ISSUE: committed the constitutional violation and thus
WON Helmut Muller is entitled to the courts will not afford protection to either
reimbursement of the funds used to acquire party).
the Antipolo property. On July 1, 1968, the same former owners
(Dinglasans) filed with the Court of First
HELD: Instance and action for recovery of the same
NO. Respondent was aware of the parcel of land. On Sept. 23, 1968, the heirs of
constitutional prohibition and expressly Lee Liong file with the trial. Both cases were
admitted his knowledge thereof to this elevated to the Supreme Court but were
Court. He declared that he had the Antipolo dismissed holding the suit barred by res
property titled in the name of petitioner judicata. On Sept. 7, 1993, Elizabeth Manuel-
because of the said prohibition. His attempt Lee and Pacita Yu Lee filed with the RTC,
at subsequently asserting or claiming a

MARIA ANNY G. YANONG JMC COLLEGE OF LAW- LLB 5YR PROGRAM Page 49
NATRES CASE DIGEST

Roxas City a petition for reconstitution of title of land. “If the land is invalidly transferred to an
Lot No. 398 of Capiz. alien who subsequently becomes a citizen or
They were the widows of the heirs of Lee transfers it to a citizen, the flaw in the original
Liong, the owner of the lot. Both widows transaction is considered cured and the title of
received a parcel of land through succession the transferee is rendered valid.”
from their deceased husbands. RTC ordered
the reconstitution of the lost and destroyed 55 Ramirez vs. Vda de Ramirez
certificate of said title of lot. On Jan. 25, 1995, (111 SCRA 704) ANTOPINA,
OSG filed with the Court of Appeals a petition Babielen Poliquit
for annulment of judgment of reconstitution
alleging that petitioners were not the proper
parties in the reconstitution of title, since Lee Republic vs. Hachero ( G.R. No. 200973,
Liong did not acquire title to the lot because he May 30, 2016)
was a Chinese citizen and was constitutionally Facts:
not qualified to own the subject land. CA In 1996, Amor Hachero (Hachero) filed his
decided, declaring the judgment of Free Patent Application No. 045307-969
reconstitution to be void. Hence this petition. covering Lot No. 1514, CAD-1150-D (subject
land) before the Community Environment
Issue/s: and Natural Resources Office (CENRO) of
1. WON OSG has the capacity to object the Palawan. The said application for free
reconstitution. patent was later approved by the Provincial
2. WON constitutional prohibition still exists Environment and Natural Resources Officer
considering the land is now under the (PENRO) of Palawan based on the following
ownership of Lee’s heirs thru succession. findings, among others that the land
applied for had been classified as alienable
Held: and disposable and, therefore, subject to
1. Yes. disposition under the Public Land Law;
The fact that the Court did not annul the sale of
the land t an alien did not validate the On October 15, 1998, Free Patent No.
transaction. For it was still contrary to the 045307-98-9384 was issued to Hachero and
constitutional proscription against aliens the subject land was registered under
acquiring lands of the public or private domain. Original Certificate of Title (OCT) No. E-
However, the proper party to assail the 18011 on May 7, 1999.
illegality of the transaction as was not the
parties to the transaction. The proper party to After an inspection and verification were
assail the sale is the Solicitor General. This conducted by the CENRO in 2000, it was
was what was done in this case when the discovered that the subject land, covered
Solicitor General initiated an action for by OCT No. E-18011, was still classified as
annulment of judgement of reconstitution of timberland and so not susceptible of
title. While it took the Republic more than sixty private ownership under the Free Patent
years to assert it, it is not barred from initiating provision of the Public Land Act. Hence, the
such action. Prescription never lies against the Republic filed the Complaint for the
State. Cancellation of Free Patent No. 045307-98-
2. No. 9384 and OCT No. E-18011 and for
The constitutional proscription on alien Reversion, which was docketed as Civil
ownership of lands of the public or private Case No. 3726.
domain was intended to protect lands from
falling in the hands of non- Filipinos. In this
case, however, there would be no more public The RTC rendered its decision in favor of
policy violated since the land is in the hands of Hachero on the ground that the free patent
Filipinos qualified to acquire and own such and title had already been issued after

MARIA ANNY G. YANONG JMC COLLEGE OF LAW- LLB 5YR PROGRAM Page 50
NATRES CASE DIGEST

Hachero was found to have complied with from the issuance of OCT No. E-18011 to
all the requirements; that it was the Hachero, the DENR personnel conducted
Republic itself thru the DENR-CENRO, another investigation and verification on
Coron, which brought the subject land the subject land. It would appear that they
under the operation of the Torrens System. suspected that a mistake was made in their
The CA affirmed the RTC decision. issuance of the patent as the subject land
Issue: had not been reclassified or released as
1. Won the petition for cancellation of alienable or disposable land. It remained
the title and reversion of the subject lot is plotted within the timberland classification
meritorious. zone.
2. WON the Government is estopped by
its previous acts. 2. Prescription and estoppel cannot
Ruling: lie against the State
1. Cancellation of title and reversion
proper
where there exists a mistake or oversight in At any rate, it is a time-honored principle
granting free patent over inalienable land that the statute of limitations or the lapse of
time does not run against the State.
Jurisprudence also recognizes the State's
Reversion is an action where the ultimate immunity from estoppel as a result of the
relief sought is to revert the land back to mistakes or errors of its officials and
the government under the Regalian agents.
doctrine. Considering that the land subject It must be emphasized that a certificate of
of the action originated from a grant by the title issued under an administrative
government, its cancellation therefore is a proceeding pursuant to a homestead
matter between the grantor and the grantee. patent, as in the instant case, is as
xxx. In Estate of the Late Jesus S. Yujuico indefeasible as a certificate of title issued
v. Republic (Yujuico case), reversion was under a judicial registration proceeding,
defined as an action which seeks to restore provided the land covered by said
public land fraudulently awarded and certificate is a disposable public land within
disposed of to private individuals or the contemplation of the Public Land Law.
corporations to the mass of public domain. Yet, we emphasize that our statement in the
It bears to point out, though, that the Court aforequoted case that a certificate of title
also allowed the resort by the Government issued pursuant to a homestead patent
to actions for reversion to cancel titles that becomes indefeasible after one year, is
were void for reasons other than fraud, i.e., subject to the proviso that "the land
violation by the grantee of a patent of the covered by said certificate is a disposable
conditions imposed by law; and lack of public land within the contemplation of the
jurisdiction of the Director of Lands to grant Public Land Law."
a patent covering inalienable forest land or But then again, the Court had several times
portion of a river, even when such grant in the past recognized the right of the State
was made through mere oversight.d to avail itself of the remedy of reversion in
In the case at bench, although the other instances when the title to the land is
Republic's action for cancellation of patent void for reasons other than having been
and title and for reversion was not based on secured by fraud or misrepresentation.
fraud or misrepresentation on the part of Be that as it may, the mistake or error of the
Hachero, his title could still be cancelled officials or agents of the BOL in this regard
and the subject land reverted back to the cannot be invoked against the government
State because the grant was made through with regard to property of the public
mistake or oversight. This could probably domain. It has been said that the State
be the reason why, shortly after one (1) year

MARIA ANNY G. YANONG JMC COLLEGE OF LAW- LLB 5YR PROGRAM Page 51
NATRES CASE DIGEST

cannot be estopped by the omission, purchasers for value and not privy to the
mistake or error of its officials or agents. alleged forgery, contend that the action cannot
lie against them.

58 Republic vs Umali (GR No. ISSUE:


80687, April 10, 1989) RAÑON, Rexie WON the sale itself be considered null and
Monicimpo void from the start, as the petitioner insists, so
as to make all titles derived therefrom also
FACTS: ineffectual, void ab initio.
The land in question is situated in Tanza,
Cavite, and consists of 78,865 square meters. HELD:
It was originally purchased on installment from No. Section 44 of P.D. 1529 (then Sec. 39 of
the government on July 1, 1910 by Florentina the Land Reg. Act), states that, every
Bobadilla, who allegedly transferred her rights registered owner receiving a certificate of title
thereto in favor of Martina, Tomasa, Gregorio in pursuance of a decree of registration, and
and Julio, all surnamed Cenizal, in 1922. every subsequent purchaser of registered land
Tomasa and Julio assigned their shares to taking a certificate of title for value and in good
Martina, Maria and Gregorio. In 1971, these faith, shall hold the same free from all
three assignees purportedly signed a joint encumbrances except those noted on the
affidavit which was filed with the Bureau of certificate and any of the encumbrances which
Lands to support their claim that they were may be subsisting, and enumerated in the law.
entitled to the issuance of a certificate of title Under said provision, claims and liens of
over the said land on which they said they had whatever character, except those mentioned
already made full payment. On the basis of by law as existing against the land prior to the
this affidavit, the Secretary of Agriculture and issuance of certificate of title, are cut off by
Natural Resources executed Deed No. V- such certificate if not noted thereon, and the
10910 (Sale Certificate No. 1280) on certificate so issued binds the whole world,
September 10, 1971, in favor of the said including the government. The Torrens system
affiants. Subsequently, on October 13, 1971, was adopted in this country because it was
TCT No. 55044 (replacing Bobadilla's OCT No. believed to be the most effective measure to
180) was issued by the register of deeds of guarantee the integrity of land titles and to
Cavite in favor of Maria Cenizal, Gregorio protect their indefeasibility once the claim of
Cenizal, and (in lieu of Martina Cenizal) ownership is established and recognized. If a
Rosalina Naval, Luz Naval, and Enrique Naval. person purchases a piece of land on the
assurance that the seller’s title thereto is valid,
When the complaint for reversion was filed on he should not run the risk of being told later
October 10, 1985, the registered owners of the that his acquisition was ineffectual after all.
land, following several transfers, were
Remedios Miclat under TCT No. 80392, Juan We find that the private respondents are
C. Pulido under TCT No. 80393, and Rosalina, transferees in good faith and for value of the
Luz and Enrique Naval under TCT No. 80394. subject property and that the original
They were named as defendants and asked to acquisition thereof, although fraudulent, did not
return the property to the State on the affect their own titles. These are valid against
aforestated grounds of forgery and fraud. The the whole world, including the government.
plaintiff claimed that Gregorio Cenizal having
died on February 25, 1943, and Maria Cenizal 59 Piñero vs Director of Lands (GR
on January 8, 1959, they could not have No. L-36507, 12/14/1974) PLAZA,
signed the joint affidavit dated August 9, 1971, Mariafe Manatad
on which Deed No. V-10910 (Sale Certificate
No. 1280) was based. The present holders of
the property claiming to be innocent

MARIA ANNY G. YANONG JMC COLLEGE OF LAW- LLB 5YR PROGRAM Page 52
NATRES CASE DIGEST

60 Kionisala vs. Dacut (GR No. that the parties stipulated only as to the
147379, 2/27/2002) PELAEZ, Chester Bryan existence of the map, but not as to
Pepito genuineness of truthfulness of its content.
Assuming that the map is admitted in
evidence, Espinosa’s rights over the
61 Evangelista vs. Santiago (GR No. property, which accrued in 1962, should not
157447, 4/29/2005) PASAOL, Ric be prejudiced by a subsequent
Jason Patlingrao classification by the State done in 1986, or
after 24 years.

62 Roco vs. Gimeda (GR No. L- Issue


11651, 12/27/1958) PAMISARAN, Excel Joy 1st Issue: Whether or not the State has
Gemota sufficiently proved that the property is part
of inalienable forest land at the time
Espinosa was granted the cadastral decree
63 Republic v. Hachero, G.R. No. and issued a title.
200973, [May 30, 2016]) NALLA, Glene
Alacayde 2nd Issue: WON the State may initiate an
action for Reversion over the subject land.
Held
No. The State failed to prove that the
64 Republic v. Espinosa, G.R. No. property was classified as forest land at the
186603, [April 5, 2017]) MISTERIO, John time of the grant of the cadastral decree
Kessler Sumauang and issuance of title to Espinosa.

Facts Since cadastral proceedings are governed


A cadastral decree was issued in favor of by the usual rules of practice, procedure,
Espinosa. The Original Certificate of Title and evidence, a cadastral decree and a
was issued in the name of Espinosa who certificate of title are issued only after the
later sold to Caliston which a Transfer applicant proves all the requisite
Certificate of Title was issued. jurisdictional facts-that they are entitled to
the claimed lot, that all parties are heard,
The State through Regional Executive and that evidence is considered.31 As such,
Director of the DENR filed a Complaint for the cadastral decree is a judgment which
annulment of title and reversion of land adjudicates ownership after proving these
with the RTC claiming the property is jurisdictional facts.
inalienable public land because it fell within
the timberland area. Here, it is undisputed that Espinosa was
granted a cadastral decree and was
RTC ruled in favor of the State and ordered subsequently issued OCT No. 191-N, the
reversion of the property. predecessor title of Caliston’s TCT No.
91117.
CA ruled in favor of Espinosa and found
that the State failed to prove fraud or Having been granted a decree in a cadastral
misrepresentation when she was issued the proceeding, Espinosa can be presumed to
Original Certificate of Title. It further ruled have overcome the presumption that the
that the State failed to prove that the land sought to be registered forms part of
property is forest land. The lone piece of the public domain.33 This means that
evidence consisting of LC Map No, 2978 Espinosa, as the applicant, was able to
was not authenticated pursuant to Section prove by incontrovertible evidence that the
24 Rule 132 of the Rules of Court. It noted

MARIA ANNY G. YANONG JMC COLLEGE OF LAW- LLB 5YR PROGRAM Page 53
NATRES CASE DIGEST

property is alienable and disposable 67 Mustang Lumber vs Court of


property in the cadastral proceedings. Appeals (June 18, 1996) JESURA

This is not to say, however, that the State


has no remedy to recover the property if 68 Paatvs Court of Appeals (January
indeed it is part of the inalienable lands of 10, 1997) INOK, Erick Jay Noro
the public domain. The State may still do so
through an action for reversion, as in the
present case. 69 Crescencio v. People, G.R. No.
205015, [November 19, 2014]) IMPIG,
2nd Issue: WON the State may initiate an Sychar M
action for Reversion over the subject land.

No. 70 Ruzon vs Sandiganbayan GR


186739, April 17, 2013 HONTANOSAS,
Reversion is the remedy where the State, Luzviminda Nee Corong
pursuant to the Regalian doctrine, seeks to
revert land back to the mass of the public
domain. It is proper when public land is 71 Aquino vs Municipality of Malay
fraudulently awarded and disposed of to Aklan (GR NO. 211356, Sept 29, 2014)
private individuals or corporations. GURO, Farhana

There are also instances when we granted FACTS:


reversion on grounds other than fraud, such as Boracay Island West Cove Management
when a “person obtains a title under the Public Philippines, Inc. applied for a building
Land Act which includes, by oversight, lands permit covering the construction of a three-
which cannot be registered under the Torrens storey hotel in Malay, Aklan, which is
system, or when the Director of Lands did not covered by a Forest Land Use Agreement
have jurisdiction over the same because it is of for Tourism Purposes (FLAgT) issued by
the public domain.”36 the DENR. The Municipal Zoning
Administrator denied petitioner’s
In this case, the State, through the Solicitor application on the ground that the proposed
General, alleges neither fraud nor construction site was within the “no build
misrepresentation in the cadastral proceedings zone” demarcated in Municipal Ordinance.
and in the issuance of the title in Espinosa’s Petitioner appealed the denial action to the
favor. No mistake or oversight was also found Office of the Mayor but despite follow up,
in the issuance of title. Hence, the remedy of no action was ever taken by the respondent
reversion could not be applied in this case. mayor.
A Cease and Desist Order was issued by
the municipal government, ordering the
closure and demolition of Boracay West
65 Ysmaelvs Deputy Executive Cove’s hotel.
Secretary (October 18, 1990) Contentions of West Cove:
LOZADA, Leah Amaya 1) The hotel cannot summarily be abated
because it is not a nuisance per se, given
the hundred million peso-worth of capital
66 Oposa vs. Factoran (July 30, infused in the venture.
1993) LARANAS, Rascille Mae 2) Municipality of Malay, Aklan should have
Dagoon first secured a court order before
proceeding with the demolition.

MARIA ANNY G. YANONG JMC COLLEGE OF LAW- LLB 5YR PROGRAM Page 54
NATRES CASE DIGEST

Contention of the Mayor: The demolition GR. No. 56948, Set. 30, 1987
needed no court order because the
municipal mayor has the express power Facts:
under the Local Government Code (LGC) to
order the removal of illegally constructed In 1968, respondent Martina Carantes for and
buildings in behalf of the Heirs of Saying Piraso filed an
The CA dismissed the petition. application for land registration over a parcel of
ISSUE: land situated in Tuba, Mountain Province,
Whether the judicial proceedings should containing a land area of 219.7879 hectares.
first be conducted before the LGU can
order the closure and demolition of the In 1970, the director of lands, through the
property in question. solicitor general, filed an opposition to the
RULING: application for registration stating, among
The Court ruled that the property involved others: That neither the applicant nor her
cannot be classified as a nuisance per se predecessors-in-interest possess sufficient title
which can therefore be summarily abated. to said parcel of land; neither it have been in
Here, it is merely the hotel’s particular open, continuous, exclusive, notorious
incident, its location and not its inherent possession and occupation of the land in
qualities that rendered it a nuisance. question for at least thirty years immediately
Otherwise stated, had it not been preceding the filing of the present application.
constructed in the no build zone, Boracay
West Cove could have secured the Few months after, the director of forestry also
necessary permits without issue. As such, filed an opposition to the application on the
even if the hotel is not a nuisance per se, it ground, that the whole area applied for
is still a nuisance per accidens registration is within the Central Cordillera
Generally, LGUs have no power to declare a Forest Reserve established under
particular thing as a nuisance unless such proclamation no. 217 dated February 16, 1929.
a thing is a nuisance per se. Despite the
hotel’s classification as a nuisance per After trial, a decision was rendered by the land
accidens, however, the LGU may registration court, adjudicating the parcel of
nevertheless properly order the hotel’s land to the applicants. The motion for
demolition. This is because, in the exercise reconsideration filed by the government in the
of police power and the general welfare Court of appeals however affirmed in toto the
clause, property rights of individuals may decision of the LRA.
be subjected to restraints and burdens in
order to fulfill the objectives of the Hence, this petition.
government. Moreover, the Local
Government Code authorizes city and Issue:
municipal governments, acting through
their local chief executives, to issue 1. Whether or not the land in question is
demolition orders. The office of the mayor part of the public forest within the Central
has quasi-judicial powers to order the Cordillera Forest Reserve.
closing and demolition of establishments. 2. Whether or not Carantes have
established registrable title over the land in
question.

72 Republic vs CA and Carantes GR Held:


No. L-56984, September 30, 1987)
FERNANDEZ, Pauline August 1. Yes. Proclamation no. 217 merely
Momongan declared a special forest reserve out of already
Republic vs CA and Carantes existing forest lands. The land was already

MARIA ANNY G. YANONG JMC COLLEGE OF LAW- LLB 5YR PROGRAM Page 55
NATRES CASE DIGEST

forest or timber land even before the 78 La Bugal-B’laan Tribal


proclamation. A person cannot enter into forest Association vs Ramos (12/1/2004) CAIGAS,
land and by the simple act of cultivating a Lloyd
portion of that land. The government must first
declare the forest land to be alienable and
disposable agricultural land before the year of 79 Loneyvs People (February 10,
entry, cultivation and exclusive and adverse 2006) CABANOG, Princess
possession can be counted for purposes of an
imperfect title.
80 DidipioEarthsavers Multipurpose
2. No. It is a well settled rule that forest Association vs. Gozun (GR No. 157882,
lands or forest reserves are not capable of 3/30/2006) CABALLERO, Jeremiah
private appropriation and possession thereof, Napalan
however long, will not convert them into private
property. The Court of Appeals finding is based
on a wrong concept of what is forest land. 81 (Southeast Mindanao Gold
There is a big difference between “forest” as Mining Corp. v. Balite Portal Mining
defined in a dictionary and “forest or timber Cooperative, G.R. No. 135190, [April 3,
land” as a classification of lands of the public 2002], 429 PHIL 668-685) BENITEZ,
domain in the Constitution. One is descriptive Winnie L
of what appears on the land while the other is a
legal status a classification for legal purposes.
82 YINLU BICOL MINING
CORPORATION vs. Trans-asia Oil and energy
73 Yngson vs Secretary of Development Corporation, GR NO. 207942,
Agriculture GR No. L-36847, July 20, 1983) January 12, 2015) BALUCANAG, April Gem
ENERO, Jomari Ivan Tagud Balansag

74 Director of Forestry vs VIllareal 83 Atok Big-Wedge Mining Co. vs


GR No. L-46048 November 29, 1988 IAC (GR No. 63528, September 9, 1996)
DALISAY, Armando, Jr. D APURADA, Kathryne Vencio

75 International Hardwood vs UP Gr 84 Apex Mining Corporation vs


No. 521518 Aug 13, 1991 CLARK, Southeast Mindanao Gold Mining Corporation
Immaculate Gonzales (GR NO 152613, June 13, 2006) ANTOPINA,
Babielen Poliquit

76 People vs Que GR No. 120365,


December 17, 1996 CEBALLOS, Jesus 85 PNOC Energy Development vs
Cadavez Veneracion (GR NO. 129820, November 30,
2006) TAN, Cesnee Joyce V.

77 Miners Association of the


Philippines vsFactoran (240 SCRA 100) 86 Benguet Corporation vs Leviste
CAMASURA, Jayson Ug-ay (GR NO. 65021, November 21, 1991)
TACDER, Lovella Fe Madelo

MARIA ANNY G. YANONG JMC COLLEGE OF LAW- LLB 5YR PROGRAM Page 56
NATRES CASE DIGEST

87 Industrial Enterprises Inc vs 96 Henares, Jr. v. Land


Court of Appeals, GR NO. 88550, April 18, Transportation Franchising and Regulatory
1990) RAÑON, Rexie M. Board, G.R. No. 158290, [October 23, 2006],
535 PHIL 835-849) JESURA

88 Sta. Rosa Realty Development


Corp. v. Court of Appeals, G.R. No. 112526, 97 People vs Que GR No. 120365,
[October 12, 2001], 419 PHIL 457-480) December 17, 1996 INOK, Erick Jay
PLAZA, Mariafe Manatad Noro

89 (National Water Resources Board 98 Cudia vs. Court of Appeals 284


v. A. L. Ang Network, Inc., G.R. No. 186450, SCRA 173 (1998) IMPIG, Sychar M
[April 8, 2010], 632 PHIL 22-32) PELAEZ,
Chester Bryan Pepito
99 Pp. vs. RosalindoCutamora, Et.
Al G.R. Nos. 133448-53. October 6, 2000
90 (Metropolitan Manila HONTANOSAS, Luzviminda Nee
Development Authority v. Concerned Corong
Residents of Manila Bay, G.R. Nos. 171947-
48, [December 18, 2008], 595 PHIL 305-352)
PASAOL, Ric Jason Patlingrao 100 People vs. Robert P. Balao, Et Al G.R.
No. 176819, Jan 26, 2011 GURO, Farhana

91 MMDA vsJancom ([G.R. No.


147465. January 30, 2002] PAMISARAN, 101 Casupanan vs. Laroya, August 26, 2002
Excel Joy Gemota FERNANDEZ, Pauline August Momongan

92 Mead vsArgel (G.R. No. L-41958, 102 Pp. vs. Sucro March 18, 1991, 195
[July 20, 1982], 115 SCRA 256) NALLA, SCRA 388 DALISAY, Armando, Jr. D
Glene Alacayde

103 People vs. Evaristo 216 S 431


93 PAB vs. Court of Appeals (G.R. CLARK, Immaculate Gonzales
No. 93891March 11, 1991195 SCRA 112)
MISTERIO, John Kessler Sumauang
104 Pp. vs. Joselito del Rosario April 14,
1999 CEBALLOS, Jesus Cadavez
94 Technology Developers, Inc. v.
Court of Appeals, G.R. No. 94759, [January
21, 1991], 271 PHIL 148-154) 105 People vs Del Rosario GR 182601
LOZADA, Leah Amaya CAMASURA, Jayson Ug-ay

95 Rodriguez, Jr. v. Intermediate 106 People vs Gerente G.R. No. 95847-48.


Appellate Court, G.R. No. 74816, [March 17, CABANOG, Princess
1987], 232 PHIL 562-575) LARANAS,
Rascille Mae Dagoon
107 People vs Tonog G.R. No. 94533
CABALLERO, Jeremiah Napalan

MARIA ANNY G. YANONG JMC COLLEGE OF LAW- LLB 5YR PROGRAM Page 57
NATRES CASE DIGEST

119 Myla Paredes Et. Al. vs. Judge Jacinto


Manalo May 10, 1995 NALLA, Glene
108 People vs. Manlulu April 22, 1994 Alacayde
BENITEZ, Winnie L

120 Abraham Ramirez vs. Antonia Corpuz-


109 Paderanga v. Court of Appeals 247 Macandong Sept. 26, 1986 MISTERIO,
SCRA 741 (1995) BALUCANAG, John Kessler Sumauang
April Gem Balansag

121 Pp. vs. Omar Mapalao May 14, 1991


110 Enrile vs Sandiganbayan G.R. No. LOZADA, Leah Amaya
213847 August 18, 2015 APURADA,
Kathryne Vencio
122 People vs. Joven De Grano G.R. No.
167710, June 5, 2009 LARANAS,
111 Borinaga v. Tamin 26 SCRA 206 (1993); Rascille Mae Dagoon
ANTOPINA, Babielen Poliquit

123 PilarCañeda Braga v. Hon. Joseph


112 Cardines v. Rozete, 242 SCRA 557 Emilio A. Abaya (GR No. 223076, september
(1995). TAN, Cesnee Joyce V. 13, 2016 JESURA

113 Pp. vs. Martin Villarama June 23, 1992 124 Hon. Ramon Jesus P. Paje, v. Hon.
TACDER, Lovella Fe Madelo Teodoro a. Casiño (G.r. No. 207257, February
03, 2015) INOK, Erick Jay Noro

114 Abubakar vs. Abubakar Oct 22, 1999


RAÑON, Rexie M.

125 Lnl Archipelago Minerals, Inc. vs.


115 Abay, Sr. vs. Garcia 162 SCRA 665; Agham party list (G.r. No. 209165)
Marcelino Rivera, Jr. vs. Pp August 30, 1990 IMPIG, Sychar M
PLAZA, Mariafe Manatad

126 Segovia v. Climate Change


116 Pp vs. Marcelino A. Bugarin (June 13, Commission, G.R. No. 211010, [March 7,
1997) PELAEZ, Chester Bryan Pepito 2017]) HONTANOSAS, Luzviminda
Nee Corong

117 Pp. vs. ElegioNadera (Feb. 2, 2000)


PASAOL, Ric Jason Patlingrao 127 Arigo v. Swift, G.R. No. 206510,
[September 16, 2014]) GURO, Farhana

118 Eligio Madrid vs. CA (May 31, 2000)


PAMISARAN, Excel Joy Gemota 128 (Resident Marine Mammals of the
Protected Seascape Tañon Strait v. Reyes,
G.R. Nos. 180771 & 181527, [April 21, 2015])
FERNANDEZ, Pauline August
Momongan

MARIA ANNY G. YANONG JMC COLLEGE OF LAW- LLB 5YR PROGRAM Page 58
NATRES CASE DIGEST

MARIA ANNY G. YANONG JMC COLLEGE OF LAW- LLB 5YR PROGRAM Page 59

You might also like